Sie sind auf Seite 1von 60

Geometry Test

2004 Rice Math Tournament


February 28, 2004
1. In the diagram below, the outer circle has radius 3, and the inner circle has radius 2. What is the area
of shaded region?

2. A parallelogram has 3 of its vertices at (1, 2), (3,8), and (4, 1). Compute the sum of the possible
x-coordinates for the 4th vertex.
ACBC
3. AC is 2004. CD bisects angle C. If the perimeter of ABC is 6012, find ADBD .

A
D B

4. P is inside rectangle ABCD. P A = 2, P B = 3, and P C = 10. Find P D.


5. Find the area of the region of the xy-plane defined by the inequality |x| + |y| + |x + y| 1.
6. We inscribe a square in a circle of radius 1 and shade the region between them. Then we incribe
another circle in the square and another square in the new circle and shade the region between the
new circle and square. After we have repeated this process infinitely many times, what is the area of
the shaded region?

[ADP ]
7. Yet another trapezoid ABCD has AD parallel to BC. AC and BD intersect at P . If [BCP ] = 12 , find
[ADP ]
[ABCD] . (Here the notation [P1 Pn ] denotes the area of the polygon P1 Pn .)

8. A triangle has side lengths 18, 24, and 30. Find the area of the triangle whose vertices are the incenter,
circumcenter, and centroid of the original triangle.

1
9. Given is a regular tetrahedron of volume 1. We obtain a second regular tetrahedron by reflecting the
given one through its center. What is the volume of their intersection?
10. Right triangle XY Z has right angle at Y and XY = 228, Y Z = 2004. Angle Y is trisected, and the
angle trisectors intersect XZ at P and Q so that X, P, Q, Z lie on XZ in that order. Find the value
of (P Y + Y Z)(QY + XY ).

2
Geometry Test
2005 Rice Math Tournament
February 26, 2005
1. A dog is tied via a 30 ft. leash to one corner of a 10 ft. by 20 ft. dog pen. Given that the dog is
initially on the outside of the pen and that neither he (nor his leash) can cross the pens fence, what
area of land does he have to roam in?
2. Find the area of the shaded region:

3. An infinite series of similar right triangles converges to point C. If AE = 16, and ED = 8, what is the
sum of all the vertical segiments (AE + BD + ...)?

4. Find the center of the circle of radius 1 centered on the y-axis that is tangent to the parabola y = x2
in two places.
5. A regular hexagon is inscribed in a circle with radius r. Another regular hexagon is formed with 2
vertices on an edge of the first hexagon and 2 vertices on the circle as shown below. Find the ratio of
the area of the smaller hexagon to the area of the larger hexagon.

1
6. In circle O, chords AB and AC are drawn so that AB = AC. Chord MN is drawn intersecting AB
and AC at points R and S, respectively. Given that AR = SC = 17 and RB = AS = 13, what is the
maximum value of |M R N S|?
7. Suppose 4ABC is an equilateral triangle with area 1. Points Q and P are on AB and points R and
S are on AC with QR, P S and BC all parallel to each other. Also, QR < P S. Points P 0 and S 0 are
chosen on BC so that P P 0 and SS 0 are each perpendicular to BC. Likewise, Q0 and R0 are chosen on
P S so that QQ0 and RR0 are perpendicular to P S. What is the maximum possible area enclosed by
the union of the two rectangles P P 0 S 0 S and QQ0 R0 R?
8. A unit lattice square in the plane is a square of side 1 whose vertices have integer coordinates. Given
that (20.05)2 1260. Let N be the number of lattice squares that are entirely contained in a circle
N
of radius 20.05 centered at the origin. Find b 10 c, where bxc is the greatest integer less than or equal
to x. (Hint: A very good approximation for area in the plane of a smooth figure is given by I + E2
where I is the number of unit lattice squares contained in the figure and E is the number intersected
by the boundary)

9. For each created by line OA and the x-axis, point X is the point (2) from AB, where OA = 6,
and AX = 3. Find the area enclosed by X as A takes each point along the unit circle.

10. An internal diagonal of a rectangular prism connects 2 vertices and does not lie entirely in one face.
Let P be a rectangular prism with volume 1, and let l be one of its internal diagonals. Suppose C is a
vertex of the prism that is not one of the vertices of l. A point K is chosen on l, and a new prism p0 is
formed such that CK is an internal diagonal of p0 , and the faces of p0 are parallel to those of p. What
is the maximum volume of p0 ?

2
Geometry Test
2006 Rice Math Tournament
February 25, 2006
1. Given a cube, determine the ratio of the volume of the octahedron formed by connecting the centers
of each face of the cube to the volume of the cube.
2. Given square ABCD of side length 1, with E on CD and F in the interior of the square so that
EF DC and AF = BF = EF , find the area of the quadrilateral ADEF .

3. Circle is centered at (0, 3) with radius 1. Circle is externally tangent to circle and tangent to the
x-axis. Find an equation, solved for y if possible, for the locus of possible centers (x, y) of circle .
4. The distance AB is l. Find the area of the locus of points X such that 15o 6 AXB 30o and X is
on the same side of line AB as a given point C.
5. Let S denote a set of points (x, y, z). We define the shadow of S to be the set of points (x, y) for which
there exists a real number z such that (x, y, z) is in S. For example, the shadow of a sphere with radius
r centered on the z axis is a circle in the xy plane centered atthe origin with radius r. Suppose a cube
has a shadow consisting of a regular hexagon witih area 147 3. What is the side length of the cube?
6. As shown below, A circle of radius R is placed tangent to two perpendicular rays. Another circle is
placed tangent to the same two rays and the first circle. Another circle is placed tangent to one ray
and tangent to both circles. What is the radius of the smallest circle, with respect to R? (The problem
was slightly rephrased and the illustration added to remove an unintended extra, simpler solution)

7. A certain 2 by 1 pool table has pockets, denoted [A, . . . , F ] as shown. A pool player strikes a ball
at point x, 14 of the way up side AC, aiming for a point 1.6 up the opposite side of the table. He
makes his mark, and the ball ricochets around the edges of the table until it finally lands in one of the
pockets. How many times does it ricochet before it falls into a pocket, and which pocket? Write your
answer in the form {C, 2006}.

1
2
8. In triangle 4P QR, the altitudes from P, Q and R measure 5, 4 and 4, respectively. Find QR .

9. Poles A, B, and P1 , P2 , P3 ,. . . are vertical line segments with bases on the x-axis. The tops of poles A
and B are (0,1) and (200,5), respectively. A string S connects (0,1) and (200,0) and intersects another
string connecting (0,0) and (200,5) at point T. Pole P1 is constructed with T as its top point. For each
integer i > 1, pole Pi is constructed so that its top point is the intersection of S and the line segment
connecting the base of Pi1 (on the x-axis) and the top of pole B. Find the height of pole P100 .
BQ
10. In triangle 4ABC, points P, Q and R lie on sides AB, BC and AC, respectively, so that PAP
B = QC =
CR 1
RA = 3 . If the area of 4ABC is 1, determine the area of the triangle formed by the points of
intersection of lines AQ, BR and CP .

2
Geometry Test
2007 Rice Math Tournament
February 24, 2007
1. An equilateral triangle has perimeter numerically equal to its area, which is not zero. Find its side
length.
2. Two spheres of radius 2 pass through each others center. Find the surface area of the regular octahe-
dron inscribed within the space enclosed by both spheres.

3. Cumulation of a polyhedron means replacing each face with a pyramid of height h using the face as a
base. There is a cumulation of the cube of side length s which (after removing unecessary edges) has
twelve sides, each a congruent rhombus. What is the height h used in this cumulation?
4. Nathan is standing on vertex A of triangle ABC, with AB = 3, BC = 5, and CA = 4. Nathan walks
according to the following plan: He moves along the altitude-to-the-hypotenuse until he reaches the
hypotenuse. He has now cut the original triangle into two triangles; he now walks along the altitude
to the hypotenuse of the larger one. He repeats this process forever. What is the total distance that
Nathan walks?
5. Given an octahedron with every edge of length s, what is the radius of the largest sphere that will fit
in this octahedron?
6. Let RICE be a quadrilateral with IE = 8, IC = 4, m6 R = 30 , m6 CER = 60 , and m6 RIE =
m6 ICE. Find CE.
7. Two congruent, regular tetrahedra of edge length 2 are positioned such that their heights (vertex to
center of base) coincide but the tetrahedra themselves do not. Find the volume of the region in which
they overlap.
8. ABC has AB = AC. Points M and N are midpoints of AB and AC, respectively. The medians
AB 2
M C and N B intersect at a right angle. Find ( BC ) .

S, T lie in the pane with S on P R and R on QT . If P Q = 5, P S = 3, P R = 5, QS = 3,


9. Points P, Q, R,
and RT = 4/ 3, what is ST ?
10. A car starts moving at constant speed at the origin facing in the positive y-direction. Its minimum
turning radius is such that it the soonest it can return to the x-axis is after driving a distance d. Let
be the boundary

of the region the car can reach by driving at most a distance d; find an x > 0 so
d d 3
that x, 3 + 2 is on .
RMT 2008 Geometry Test February 23, 2008

1. A regular polygon of side length 1 has the property that if regular pentagons of side length 1 are placed
on each side, then each pentagon shares a side with the two adjacent ones. How many sides does such
a polygon have?
2. John stands against one wall of a square room with walls of length 4 meters each. He kicks a frictionless,
perfectly elastic ball in such a way that it bounces off the three other walls once each and returns to
him (diagram not geometrically accurate). How many meters does the ball travel?

3. A cube is inscribed in a sphere of radius r. Find the ratio of the volume of the sphere to that of the
cube.
4. A circle of radius 144 has three smaller circles inside it, all congruent. Each small circle is tangent to
the other two and to the large circle. Find the radius of one of the smaller circles.

5. In 4ABC, C is right, AC = 2 3 + x and BC = 1 2x + x 3. Find mB.
6. Points A, B, C lie on sides DE, EF , and F E of 4DEF , respectively. If DA = 3, AE = 2, EB = 2,
BF = 11, F C = 11, and CD = 1, find the area of 4ABC.
7. What is the area of the incircle of a triangle with side lengths 10040, 6024, and 8032?

8. Rhombus ABCD has side length l, with cos(mB) = 23 . The circle through points A, B, and D has
radius 1. Find l.
9. A trapezoid has bases of length 10 and 15. Find the length of the segment that stretches from one leg
of the trapezoid to the other, parallel to the bases, through the intersection point of the diagonals.

10. A regular polygon with 40 sides, all of length 1, is divided into triangles, with each vertex of each
triangle being a vertex of the original polygon. Let A be the area of the smallest triangle. What is the
minimum number of square root signs needed to express the exact value of A?
RMT 2009 Geometry Test February 21, 2009

1. The sum of all of the interior angles of seven polygons is 180 17. Find the total number of sides of the
polygons.
2. The pattern in the figure below continues inward infinitely. The base of the biggest triangle is 1. All
triangles are equilateral. Find the shaded area.

3. Given a regular pentagon, find the ratio of its diagonal, d, to its side, a.
4. ABCD form a rhobus. E is the intersection of AC and BD. F lie on AD such that EF F D. Given
EF = 2 and F D = 1. Find the area of the rhobus ABCD.

5. In the 2009 Rice Olympics, Willy and Sammy are two bikers. The circular race track has two lanes,
the inner lane with radius 11, and the outer with radius 12. Willy will start on the inner lane, and
Sammy on the outer. They will race for one complete lap, measured by the inner track. What is the
square of the distance between Willy and Sammys starting positions so that they will both race the
same distance? Assume that they are of point size and ride perfectly along their respective lanes.
6. Equilateral triangle ABC has side length of 24. Points D, E, F lie on sides BC, CA, AB such that
AD BC, DE AC, and EF AB. G is the intersection of AD and EF . Find the area of the
quadrilateral BF GD.
7. Four disks with disjoint interiors are mutually tangent. Three of them are equal in size and the fourth
one is smaller. Find the ratio of the radius of the smaller disk to one of the larger disks.

8. Three points are randomly placed on a circle. What is the probability that they lie on the same
semicircle?
9. Two circles with centers A and B intersect at points X and Y . The minor arc XY = 120 with
respect to circle A, and XY = 60 with respect to circle B. If XY = 2, find the area shared by the
two circles.

10. Right triangle ABC is inscribed in circle W . CAB = 65 , and CBA = 25 . The median from C
to AB intersects W at D. Line l1 is drawn tangent to W at A. Line l2 is drawn tangent to W at D.
The lines l1 and l2 intersect at P . Compute AP D.
RMT 2010 Geometry Test February 20, 2010

Note: Figures may not be drawn to scale.


1. Find the reflection of the point (11, 16, 22) across the plane 3x + 4y + 5z = 7.
2. Given the three points (1608, 2010, 2010), (2010, 2412, 2010), and (2010, 2010, 2412). Find the area of
the circle defined by these three points.
3. What is the inradius of a triangle with side lengths 4, 5, and 6?
4. Find the volume of a regular cubeoctahedron, of sidelength 1, which is a solid of 8 equilateral triangles
and 6 squares such that each edge is a square and a triangle together, as pictured.

5. Given triangle ABC. D lies on BC such that AD bisects BAC. Given AB = 3, AC = 9, and
BC = 8. Find AD.
6. Given the information in the diagram, let M BD = 90 , OT = 25 and AM = M B = 30. Find M D.
M

A
B

T
O

7. Suppose we have a polyhedron consisting of triangles and quadrilaterals, and each vertex is shared by
exactly 4 triangles and one quadrilateral. How many vertices are there?
8. Given the following circular section, write the height h, the height of the circle above the x-axis at a
given x, as a function of x, with R x R. (Note and R are constants and is the angle between
the x-axis and the tangent line to the circle at x = R.)

9. A sphere of radius 1 is internally tangent to all four faces of a regular tetrahedron. Find the tetrahe-
drons volume.
10. We are given a coin of diameter 12 and a checkerboard of 1 1 squares of area 2010 2010. We must
toss the coin such that it lands completely on the checkerboard. If the probability that the coin doesnt
2
touch any of the lattice lines is ab2 where ab is a reduced fraction, find a + b.
RMT 2011 Geometry Test February 19, 2011

1. Jeffrey starts out at (0, 0) facing in some direction. Each second, Jeffrey walks forward 1 unit, and
then turns counterclockwise by 45 . When Jeffrey returns to his starting point, what is the area of the
shape he has made.
2. Pentagon ABCDE is inscribed in a circle of radius 1. If DEA
= EAB
= ABC, mCAD = 60 ,
and BC = 2DE, compute the area of ABCDE.

3. Let circle O have radius 5 with diameter AE. Point F is outside circle O such that lines F A and F E

intersect circle O at points B and D, respectively. If
F A =10 and
mF AE = 30 , then the perimeter
of quadrilateral ABDE can be expressed as a + b 2 + c 3 + d 6, where a, b, c, and d are rational.
Find a + b + c + d.
4. Let 4ABC be equilateral. Two points D and E are on side BC (with order B, D, E, C), and satisfy
DAE = 30 . If BD = 2 and CE = 3, what is BC?
A

30

B 2 D E 3 C

5. Let ABCD be a cyclic quadrilateral with AB = 6, BC = 12, CD = 3, and DA = 6. Let E, F be the


intersection of lines AB and CD, lines AD and BC respectively. Find EF .
6. Two parallel lines l1 and l2 lie on a plane, distance d apart. On l1 there are an infinite number of
points A1 , A2 , A3 , , in that order, with An An+1 = 2 for all n. On l2 there are an infinite number of
points B1 , B2 , B3 , , in that order and in the same direction, satisfying
P Bn Bn+1 = 1 for all n. Given
that A1 B1 is perpendicular to both l1 and l2 , express the sum i=1 Ai Bi Ai+1 in terms of d.

7. In an unit square ABCD, find the minimum of 2AP + BP + CP where P is a point inside ABCD.
8. We have a unit cube ABCDEF GH where ABCD is the top side and EF GH is the bottom side with
E below A, F below B, and so on. Equilateral triangle BDG cuts out a circle from the cubes inscribed
sphere. Find the area of the circle.
9. Given 4ABC. Let A0 lie on BC such that BA0 = 41 A0 C, B 0 lie on AC such that AB 0 = B 0 C, and C 0
lie on AB such that 2AC 0 = BC 0 . Let D be the point of intersection between AA0 and CC 0 , E the
point of intersection between AA0 and BB 0 , and F the point of intersection between BB 0 and CC 0 .
What is area(4DEF )
area(4ABC) ?

10. Given a triangle ABC with a = 5, b = 7, and c = 8, find the side length of the largest equilateral
triangle P QR such that A, B, C lie on QR, RP, P Q respectively.
RMT 2012 Geometry Test February 18, 2012

_ _
1. AB is a diameter of a circle with radius 1. C lies on this circle such that AC / BC= 4. Find the (positive)
_ _
difference in area between AC CA, the segment of the circle cut off by AC, and BC CB, the segment
cut off by BC.

2. In trapezoid ABCD, BC k AD, AB = 13, BC = 15, CD = 14, and DA = 30. Find the area of ABCD.
3. Let ABC be equilateral triangle of side 1. Draw three circles Oa , Ob , Oc with diameters BC, CA, and
AB, respectively. Let Sa denote the area of the region inside Oa and outside of Ob and Oc . Define Sb and
Sc similarly, and let S be the area of the intersection between the three circles. Find Sa + Sb + Sc S.
4. Let ABCD be a rectangle with area 2012. There exist points E on AB and F on CD such that DE =
EF = F B. Diagonal AC intersects DE at X and EF at Y . Compute the area of triangle EXY .
5. What is the radius of the largest sphere that fits inside an octahedron of side length 1?
6. A red unit cube ABCDEF GH (with E below A, F below B, etc.) is pushed into the corner of a room
with vertex E not visible, so that faces ABF E and ADHE are adjacent to the wall and face EF GH is
adjacent to the floor. A string of length 2 is dipped in black paint, and one of its endpoints is attached to
vertex A. How much surface area on the three visible faces of the cube can be painted black by sweeping
the string over them?
7. Let ABC be a triangle with side lengths 5, 8, and 9 and incircle O. Consider the other tangent line to O
parallel to BC, which intersects AB at Ba and AC at Ca . Let ra be the inradius of triangle ABa Ca , and
define rb and rc similarly. Find ra + rb + rc .
8. In triangle ABC, we have a = 3, b = 5, and c = 6. For some radius r, draw the three circles with radius
r such that the centers Oa and Ob are outside the triangle while the center Oc is inside the triangle. If
circles Oa and Ob intersect at C, Ob and Oc intersect at A, Oc and Oa intersect at B, and all three circles
intersect at a fourth distinct point, find r.

9. In quadrilateral ABCD, ABD = BCD and ADB = ABD + BDC. If AB = 8, and AD = 5,


find BC.
p
10. A large flat plate of glass is suspended 2/3 units above a large flat plate of wood.
(The glass is infinitely
thin and causes no funny refractive effects.) A point source of light is suspended 6 units above the glass
plate. An object rests on the glass plate of the following description. Its base is an isosceles trapezoid
ABCD with ABkDC, AB = AD = BC = 1, and DC = 2. The point source of light is directlyabove
the midpoint of CD. The objects upper face is a triangle EF G with EF = 2, EG = F G = 3. G
and AB lie on opposite sides of the rectangle EF CD. The other sides of the object are EA = ED = 1,
F B = F C = 1, and GD = GC = 2. Compute the area of the shadow that the object casts on the wood
plate.
RMT 2013 Geometry Test February 2, 2013

1. In triangle ABC, AC = 7. D lies on AB such that AD = BD = CD = 5. Find BC.

2. What is the perimeter of a rectangle of area 32 inscribed in a circle of radius 4?

3. Robin has obtained a circular pizza with radius 2. However, being rebellious, instead of slicing
the pizza radially, he decides to slice the pizza into 4 strips of equal width both vertically and
horizontally. What is the area of the smallest piece of pizza?

4. ABCD is a regular tetrahedron with side length 1. Find the area of the cross section of ABCD
cut by the plane that passes through the midpoints of AB, AC, and CD.

5. In square ABCD with side length 2, let P and Q both be on side AB such that AP = BQ = 12 .
Let E be a point on the edge of the square that maximizes the angle P EQ. Find the area of
triangle P EQ.

6. ABCD is a rectangle with AB = CD = 2. A circle centered at O is tangent to BC, CD, and


AD (and hence has radius 1). Another circle, centered at P , is tangent to circle O at point T
and is also tangent to AB and BC. If line AT is tangent to both circles at T , find the radius of
circle P .

7. ABCD is a square such that AB lies on the line y = x + 4 and points C and D lie on the graph
of parabola y 2 = x. Compute the sum of all possible areas of ABCD.

8. Let equilateral triangle ABC with side length 6 be inscribed in a circle and let P be on arc AC
such that AP P C = 10. Find the length of BP .

9. In tetrahedron ABCD, AB = 4, CD = 7, and AC = AD = BC = BD = 5. Let IA , IB , IC ,


and ID denote the incenters of the faces opposite vertices A, B, C, and D, respecitvely. It is
provable that AIA intersects BIB at a point X, and CIC intersects DID at a point Y . Compute
XY .

10. Let triangle ABC have side lengths AB = 16, BC = 20, AC = 26. Let ACDE, ABF G, and
BCHI be squares that are entirely outside of triangle ABC. Let J be the midpoint of EH, K
be the midpoint of DG, and L the midpoint of AC. Find the area of triangle JKL.
RMT 2014 Geometry Test February 15, 2014

Time limit: 50 minutes.


Instructions: This test contains 10 short answer questions. All answers must be expressed in sim-
plest form unless specified otherwise. Only answers written on the answer sheet will be considered
for grading.
No calculators.

1. The coordinates of three vertices of a parallelogram are A(1, 1), B(2, 4), and C(5, 1). Compute
the area of the parallelogram.

2. In a circle, chord AB has length 5 and chord AC has length 7. Arc AC is twice the length of
arc AB, and both arcs have degree less than 180. Compute the area of the circle.

3. Spencer eats ice cream in a right circular cone with an opening of radius 5 and a height of 10.
If Spencers ice cream scoops are always perfectly spherical, compute the radius of the largest
scoop he can get such that at least half of the scoop is contained within the cone.

4. Let ABC be a triangle such that AB = 3, BC = 4, and AC = 5. Let X be a point in the


triangle. Compute the minimal possible value of AX 2 + BX 2 + CX 2 .

5. Let ABC be a triangle where BAC = 30 . Construct D in 4ABC such that ABD =
ACD = 30 . Let the circumcircle of 4ABD intersect AC at X. Let the circumcircle of
4ACD intersect AB at Y . Given that DB DC = 10 and BC = 20, find AX AY .

6. Let E be an ellipse with major axis length 4 and minor axis length 2. Inscribe an equilateral
triangle ABC in E such that A lies on the minor axis and BC is parallel to the major axis.
Compute the area of 4ABC.

7. Let ABC be a triangle with AB = 13, BC = 14, and AC = 15. Let D and E be the feet of the
altitudes from A and B, respectively. Find the circumference of the circumcircle of 4CDE.

8. O is a circle with radius 1. A and B are fixed points on the circle such that AB = 2. Let C
be any point on the circle, and let M and N be the midpoints of AC and BC, respectively. As
C travels around circle O, find the area of the locus of points on M N .

9. In cyclic quadrilateral ABCD, AB


= AD. If AC = 6 and AB
BD = 35 , find the maximum possible
area of ABCD.

10. Let ABC be a triangle with AB = 12, BC = 5, AC = 13. Let D and E be the feet of the
internal and external angle bisectors from B, respectively. (The external angle bisector from B
bisects the angle between BC and the extension of AB.) Let be the circumcircle of 4BDE;
extend AB so that it intersects again at F . Extend F C to meet again at X, and extend
AX to meet again at G. Find F G.
RMT 2015 Geometry Test February 14, 2015

Time limit: 50 minutes.


Instructions: This test contains 10 short answer questions. All answers must be expressed in
simplest form unless specified otherwise. Only answers written inside the boxes on the answer
sheet will be considered for grading.
No calculators.

1. Clyde is making a Pacman sticker to put on his laptop. A Pacman sticker is a circular sticker
of radius 3 inches with a sector of 120 cut out. What is the perimeter of the Pacman sticker in
inches?

2. In a certain right triangle, dropping an altitude to the hypotenuse divides the hypotenuse into
two segments of length 2 and 3 respectively. What is the area of the triangle?

3. Consider a triangular pyramid ABCD with equilateral base ABC of side length 1. AD = BD =
CD and ADB = BDC = ADC = 90 . Find the volume of ABCD.

4. Two circles with centers A and B respectively intersect at two points C and D. Given that
A, B, C, D lie on a circle of radius 3 and circle A has radius 2, what is the radius of circle B?

5. Consider two concentric circles of radius 1 and 2. Up to rotation, there are two distinct equilateral
triangles with two vertices on the circle of radius 2 and the remaining vertex on the circle of
radius 1. The larger of these triangles has sides of length a, and the smaller has sides of length
b. Compute a + b.

6. In a triangle ABC, let D and E trisect BC, so BD = DE = EC. Let F be the point on AB
AF AG 1
such that = 2, and G on AC such that = . Let P be the intersection of DG and EF ,
FB GC 2
BX
and extend AP to intersect BC at a point X. Find .
XC
7. A unit sphere is centered at (0, 0, 1). There is a point light source located at (1, 0, 4) that sends
out light uniformly in every direction but is blocked by the sphere. What is the area of the
spheres shadow on the x-y plane? (A point (a, b, c) denotes the point in three dimensions with
x-coordinate a, y-coordinate b, and z-coordinate c).

8. Consider the parallelogram ABCD such that CD = 8 and BC = 14. The diagonals AC and
BD intersect at E and AC = 16. Consider a point F on the segment ED with F D = 366 .
Compute CF .

9. Triangle ABC is isoceles with AB = AC = 2 and BC = 1. Point D lies on AB such that the
inradius of ADC equals the inradius of BDC. What is the inradius of ADC?

10. For a positive real number k and an even integer n 4, the k-Perfect n-gon is defined to be the
equiangular n-gon P1 P2 . . . Pn with Pi Pi+1 = Pn/2+i Pn/2+i+1 = k i1 for all i {1, 2, . . . , n/2},
assuming the convention Pn+1 = P1 (i.e. the numbering wraps around). If a(k, n) denotes the
a(2,24)
area of the k-Perfect n-gon, compute a(4,12) .
Geometry Solutions
2004 Rice Math Tournament
February 28, 2004

1. Answer: 4 2
Consider the following diagram, where O is the center of the larger circle: Since the smaller circle has
Q

O P x

radius 2, we must have x = 2 and OP = 1. And OQ is a radius of the larger circle, so OQ = 3.


Applying the Pythagorean theorem to triangle OP Q, we find that

12 + y 2 = 3 2 .

Solving this for y gives us y = 2 2.
The shaded region is a triangle with base 2y and height x. Therefore its area is
1
(x)(2y) = xy = 4 2.
2

2. Answer: 8
There are 3 possible locations for the 4th vertex. Let (a, b) be its coordinates. If it is opposite to
vertex (1, 2), then since the midpoints of the diagonals of a parallelogram coincide, we get ( a+1 b+2
2 , 2 )=
3+4 8+1
( 2 , 2 ). Thus (a, b) = (6, 7). By similar reasoning for the other possible choices of opposite vertex,
the other possible positions for the fourth vertex are (0, 9) and (2, 5), and all of these choices do give
parallelograms. So the answer is 6 + 0 + 2 = 8.
3. Answer: 4
1 1
Let AC = d, BC = c, AD = a, BD = b so ad = bc . Now a+b a b a+b b
c = c + d = a( c + d ), d = c + d =
b
1 1 c+d 1 1 1 cd a+b c+d 2 cd
b( c + d ), a+b = 1 + 1 ( a + b ) = c+s ab . This implies that ( a+b ) = ab . Now a + b = 2002,
c d
cd 4008 2
c + d = 6006 2002 = 4008 so ab = ( 2004 ) = 4.

4. Answer: 95
A y E z B

w w
P
H F

x x

D y G z C

Draw perpendiculars from P to E on AB, F on BC, G on CD, and H on DA, and let AH = BF = w,
HD = F C = x, AE = DG = y, and EB = GC = z. Then P A2 = w2 + y 2 , P B 2 = w2 + z 2 , P C 2 =
x2 + z 2 , and 2 2 2 2 2 2 2
P D = x + y . Adding and subtracting, we see that P D = P A P B + P C = 95,
so P D = 95.

1
3
5. Answer: 4
To graph this region we divide the xy-plane into six sectors depending on which of x, y, x + y are 0,
or 0. The inequality simplifies in each case:

Sector Inequality Simplified inequality


x 0, y 0, x + y 0 x + y + (x + y) 1 x + y 21
x 0, y 0, x + y 0 x y + (x + y) 1 x 21
x 0, y 0, x + y 0 x y (x + y) 1 y 21
x 0, y 0, x + y 0 x + y + (x + y) 1 y 12
x 0, y 0, x + y 0 x + y (x + y) 1 x 12
x 0, y 0, x + y 0 x y (x + y) 1 x + y 21

We then draw the region; we get a hexagon as shown. The hexagon intersects each region in an isosceles
right triangle of area 81 , so the total area is 6 81 = 34 .

6. Answer: 2 4

The first square has side length of 2. The second circle has half of the area of the first one. Thus
the shaded region has half of the area of the first one. The second shaded area is half of the size of the
first. But the sum of 21n from zero to infinity is 2 so the shaded area is 2( 2) = 2 4.

7. Answer: 3 2 2
A homothety (scaling) about P takes triangle ADP into BCP , since AD, BC are parallel and A, P, C;
B, P, D are collinear. The ratio
of homothety is thus 2. It follows that, if we rescale to put [ADP] = 1,
then [ABP ] = [CDP ] = 2, just by the ratios of lengths of bases. So [ABCD] = 3 + 2 2, so
[ADP ] 1

[ABCD] = 3+2 2 . Simplifying this, we get 3 2 2.

8. Answer: 3
There are many solutions to this problem, here is one. The given triangle is a right 3-4-5 triangle, so
the circumcenter is the midpoint of the hypotenuse. Coordinatizing for convenience, put the vertex at
(0, 0) and the other vertices at (0, 18) and (24, 0). Then the circumcenter is (12, 9). The centroid is
at one-third the sum of the three vertices, which is (8, 6). Finally, since the area equals the inradius
1824
times half the perimeter, we can see that the inradius is 18+24+30
2
= 6. So the incenter of the triangle
2
is (6, 6). So the small triangle has a base of length 2 and a height of 3, hence its area is 3.
1
9. Answer: 2
Imagine placing the tetrahedron ABCD flat on a table with vertex A at the top. By vectors or
otherwise, we see that the center is 34 of the way from A to the bottom face, so the reflection of this
face lies in a horizontal plane halfway between A and BCD. In particular, it cuts off the smaller
tetrahedron obtained by scaling the original tetrahedron by a factor of 21 about A. Similarly, the
reflections of the other three faces cut off tetrahedra obtained by scaling ABCD by 21 about B, C, and

2
D. On the other hand, the octahedral piece remaining remaining after we remove these four smaller
tetrahedra is in the intersection of ABCD with its reflection, since the reflection sends this piece to
itself. So the answer we seek is just the volume of this piece, which is

1
(volume of ABCD) 4 (volume of ABCD scaled by a factor of 2)
= 1 4( 21 )3 = 12 .

10. Answer: 1370736

X
P
Q

Y Z

The triangles area is 2282004


2 = 228456. All the angles at Y are 30 degrees, so by the sine area formula,
the areas of the three small triangles in the diagram are QY 4Y Z , P Y 4QY , and XY 4P Y , which sum to
the area of the triangle. So expanding (P Y + Y Z)(QY + XY ), we see that it equals

4 228456 + XY Y Z = 6 228456 = 1370736.

3
General Solutions
2005 Rice Math Tournament
February 26, 2005
1. Answer: 7
3x + 5y = 83, x + y = 23, 2y = 14, y = 7, So there are seven five legged goats.

2. Answer: 2
Its pretty obvious that a circle that intersects the lattice point vertices of a unit square will have the
optimal area. The answer is 2 .

3. Answer: 2
By definition
4. Answer: 50
Number with both = total combinations - without 2 - without 1 + 1. This is 34 24 24 + 1 = 50.
5. Answer: 71
Can be found by trial and error or using the formula m n m n for m, n relatively prime.
6. Answer: Sunday
Febuary 26, 2005 is a Saturday. It is 285 days until December 8, 2005. Since 280 is a multiple of 7,
that will be a Thursday. 2019 is 14 years from then and that will be a multiple of 7 (14 365) plus
the number of leap years which is 3. Thus December 8, 2019 will be a Sunday.
7
7. Answer: 160
If the 4-sided die shows a 1 there are 19 different rolls of the first 20-sided die that can still achieve a
sum of 21. If the 4-sided die shows a 2 there are 18. If it shows a 3 there are 17, and if a 4, there are
16. Thus, there are 19 + 18 + 17 + 16 = 70 ways to get 21.
70 7
=
20 20 4 160

8. Answer: 165
The number of 0s is the number of factors of 26 there are. As 26 = 132, and there are more factors of
13 than 2s, we need only find the number of factors of 13 in 2005!. 2005 13 > 154; 2005 169 > 11;
154 + 11 = 165
9. Answer: m2
A lillypad will be off if an even number of frogs jump on it. Hence, the mth lillypad will be on if m
has an odd number of factors. m has an odd number of factors m is a perfect square.
10. Answer: 15
All
6 such
parallelograms have 2 vertices with y > 0. There are 6 points that satisfy this so there are
2=15 ways to pick them. Each choice uniquely determines the other 2 points of the parallelogram, so
the answer is 15.
18! 18!
11. Answer: 2!2!3!
= 24
The numerator denotes the number of arrangements of 18 different words. The denominator divides
out redundancies since in, the, and words are repeated. Note from author: I am purposely not
capitalizing the first letter of the sentence as this might otherwise lead to ambiguity.

1
12. Answer: 800

3 1 1
302 + 102 + 202 = 800
4 4 4
13. Answer: 61
Of the 4 consecutive integers two must be odd and one must be of the form 4m+2. Therefore for 128
to divide the product, the remaining integer must be a multiple of 64. Hence the least such n is n=61.
14. Answer: 32
2 2 2
Since all triangles are similar, AE : EB = EB : ED. Let EB AE = x. Since EB = ED + BD ,

(16x)2 = 82 + (16x2 )2 , x = 22 . Since AE = 16, BD = 16x2 , the next vertical segment is 16x2 x2 ,
16
and so on. sum of all vertical segments is a geometric series 1x 2 = 32.

19
15. Answer: 96
Consider the result of each toss mod 5. We have a success if we roll:
3
(1, 4) with probability 12 18 = 96
3
3 2 6
(2, 3) with probability 12 8 = 96
2 2 4
(3, 2) with probability 12 8 = 96
2 2 4
(4, 1) with probability 12 8 = 96
2 1 2
(0, 0) with probability 12 8 = 96
19
The total chances are 96 .
16. Answer: The coefficient of x1000 in (1 + x2 x5 )2005
x1000 is even, so its coefficient is unchanged under the transformation x x.
Under this transformation,
(1 x2 + x5 )2005 (1 x2 x5 )2005 and
(1 + x2 x5 )2005 (1 + x2 + x5 )2005 .
After expanding (1 + x2 + x5 )2005 , the x1000 will all have positive coefficients.
However, there will be x1000 terms in the expansion of (1 x2 x5 )2005 with negative coefficients.

17. Answer: 840


There are 6! ways to order the numbers 1-6. Half will have the 1 before the 2 and half will have the
2 before the 1. Thus, the number of ways to stack the leftmost six blocks is 26!3 . Now we must simply

determine in which three of the nine positions the sevens could have been read. There are 93 ways to
do this. Thus, our final answer is 26!3 9C3 = 65432
23 987
32 = 7 6 5 4 = 840.

18. Answer: 12
There are 10 powers of 4 (1, 4, . . ., 49 ) and 8 powers of 6 (1, 6, . . ., 67 ), but we have counted powers
of 12 twice and there are 6 of these. The inclusion-exclusion principle tells us that the answer is
10 + 8 6 = 12 satisfactory numbers.
19. Answer: 625f t
ft
The walls move 40 sec total taking 25 sec. to close. Therefore, the ball travels 25 25 = 625f t.
20. Answer: 36
Let n be an integer with 1 n 100, and say n = ab where a is not a perfect power. Then
nn = (ab )n = abn . Since a is not a perfect power, if nn is a perfect cube then bn is divisible by 3, so
either b or n is divisible by 3. If b is divisible by3, n = ab is a perfect cube, s n = 1, 8, 27, and 64. If n
is divisible by 3, then n = 3, 6, 9, ..., 99. This is a total of 4+33=37 n-values, but n = 27 was counted
twice, so there are 36 perfect cubes.

2
21. Answer: 247 = 30 + 42 + 70 + 105
No set of less than four containers works, since any three of them have at least one prime factor in
common. To measure 1, we can measure 6 with the 30 and 42 containers, then 2 with the 6 we found
plus the 70, and finally 1 when we add the 105 to the mix.
3
22. Answer: 4
From case by case analysis, the sides are found to be: x + y = 12 , x + y = 12 , x = 12 , and y = 12 .
This forms a hexagon which can be split up into two squares and two isosceles right triangles, with
side/leg length 21 . Thus the area is 2( 12 )2 + 2 12 ( 12 )2 = 3( 12 )2 = 34 .
23. Answer: 3
The roots are n, n + 1, and n + 2. Then a2 = ((n) + (n + 1) + (n + 2))2 = (3n + 3)2 = 9(n + 1)2 .
a2
b + 1 = n(n + 1) + n(n + 2) + (n + 1)(n + 2) + 1 = 3n2 + 6n + 3 = 3(n + 1)2 . So b+1 = 3.
7! 4!
24. Answer: 5 4
2
= 15 7!
The orderings of the vowels and consonants are independent of how the vowels and consonants are
ordered relative to each other. So first we compute the number of ways to arrange the vowels and
consonants. Since the 4 Vs partition the 7 Cs into 5 possibly empty blocks. Since the maximum
these blocks can have to satisfy the rules are 1,2,2,2, and 1, so exactly one of them must be one short.
There are 5 ways to do this. Then we multiply the number of permutations of the vowels AEAI and
consonants MTHMTCS, of which there are 7! 4!
4 2.

25. Answer: 8

a1 = 0
1
a2 =
4
1
an = an1 +
4
1
an = (n 1)
4
1 1
(33 1) = (32) = 8
4 4

3
Geometry Solutions
2006 Rice Math Tournament
February 25, 2006
1. Answer: 16
Let s represent the side length of the cube. The octahedron has a volume
equivalent to the volume
of two pyramid with height 2s and a square base with side length 2s 2. The volume is therefore
2
2 31 2s 2 2s = 61 s3 , or 1/6 of the cube volume.
2. Answer: 13
32
Let EF = x.

From pythagorean theorem:


( 12 )2 + (1 x)2 = x2
1 + 4x2 8x + 4 = 4x2
8x = 5
x = 85
1 ( 12 )( 83 ) 1 3 13
area of ADEF = area of ADEG area of AF G = 2 2 = 2 32 = 32
2
3. Answer: y = x8 + 1
Since circle is tangent to the x-axis, its radius is y. Thus from the Pythagorean Theorem:
(3 y)2 + x2 = (y + 1)2
9 6y + x2 = 2y + 1
8 + x2 = 8y
x2
1+ =y
8
 
4. Answer: 12 + 1 + 1112 3
Since inscribed angles intercept arcs of measure twice that of the inscribed angle, this is the area above
line AB between circles centered at P and Q, with 6 AQB = 60 and 6 AP B = 30 , A, B on both
circles, and P, Q on the perpendicular bisector of AB. Let M be the midpoint of AB. 4AQB is then
equilateral, so QM = 23 , so the radius of circle Q is 1. We see that since 6 AP B = 30 , P is on circle
 2 2
Q, so P M = 1 + 23 , and by the Pythagorean theorem, (P A)2 = 1 + 23 + 12 = 2 + 3. We find
the area in circles P and Q above line AB by taking the major sector AB of each the circles above the
line and adding in the areas of 4AP B and 4AQB respectively:
! !!
  330 1 3 2 300 1 3
A= 2+ 3 + (1) 1 + 1 + (1)
360 2 2 360 2 2

1
This simplifies to the given answer.

5. Answer: 7 3
Let the side length of the cube be s. It is apparent that in order for the shadow to be a regular hexagon,
the cube must have two vertices with the same x and y coordinates; call these vertices A and B. Let T
be another vertex of the cube. Clearly, 4ABT is a right triangle with hypotenuse AB = s 3 the space
diagonal of the cube, and legs s and s 2. Notice that a segment from T to AB has for its shadow a
segment between the center of the hexagon and one of its vertices; thus the distance from T to AB is
the same as the center to vertex distance. Using similar triangles, this length can be found to be s 3 6 .

Thus the area of the hexagon is s2 3 = 147 3 and therefore s = 7 3.

6. Answer: R 2
(3 2 2)

Let
the radius of
the second circle be r.
R 2 R = r + r 2

r = R(2+1
21)
= R(3 2 2)
Let
p the radius of the third
p circle be . p
(r + )2 (r )2 + (R + )2 (R )2 = (R + r)2 (R r)2

4r + 4R = 4Rr

r + R = Rr

Rr
= R+
r
Rr R2 (32 2) R2 (32 2) R2 (32 2) R
= = = = 2R = 2 (3 2 2)
R+r+2 Rr R(42 2)+2R 32 2 R(42 2)+2R( 21)

7. Answer: {D, 6}

Note that each time the ball bounces up the wall, it is equivalent to forming a mirror image of the table
and extending the path taken. Set up sides AF and AC as the x and y coordinate axes, respectively.

2
Since the ball is hit at (0,0.5), it can travel in an imaginary straight line through imaginary images of
the table until it hits an integer coordinate (i.e. a pocket). Therefore,
11
0.5 + (1.6 1.5)x = y1 + x = 2y
5
It is clear that the first instance of integer (x, y) occurs when x = 5 and y = 6. Simply counting, 5 units
in the x direction ends up on side DF , and 6 units in the y direction would be on side CD. Therefore,
the ball must have fallen in at this intersection, into pocket D. Drawing iterations of the pool table to
fill the rectangle from (0, 0) to (5, 6), we see that the ball has crossed four vertical boundaries and two
horizontal boundaries, making 6 ricochets.
8. Answer: 400 21
A
If M is the midpoint of QR, then P M QR = 2A, where A is the area of the triangle. So QM = 5 and,
by the same logic, P Q = A2 . Use the Pythagorean Theorem on triangle 4P QM to get A = 50
21
QR = 20 .
21

1
9. Answer: 21
Arbitrarily label the heights of poles A and B as a and b, respectively. Suppose poles A and B are p
and q units, respectively, from pole P1 (as measured along the x-axis). Then the height of P1 , call it x,
q p
satisfies: xa = p+q and xb = p+q x = a+bab
. The same procedure yields the height of P2 : just replace
ab ab ab ab
a by a+b in the above equation to get 2a+b . Generalize by replacing a by na+b to get (n+1)a+b as the
1
height of Pn+1 . Now put a = 1, b = 5 and n = 100 to get 21 .
4
10. Answer: 13
Let O be the intersection of AQ and BR. Our goal is to find the area of 4ABO = 1 BQ AO 1 AO
BC AQ = 4 AQ .
Using mass points, place a mass of 1 at B and therefore a mass of 31 at C since 1BQ = 13 QC. Likewise,
vertex A bears a mass of 19 . Replace the masses at B and C with a mass of 1 + 13 = 34 at Q. Thus,
AO 4/3 AO 12 1 12 3
OQ = 1/9 = 12. Hence, AQ = 13 . Thus, the area of 4ABO = 4 13 = 13 , which is independent of
the side lengths of 4ABC. There are two additional nonoverlapping triangles like 4ABO that must
3 3 4
also have an area of 13 . The area of the central triangle is 1 3 13 = 13 .

3
General Solutions
2007 Rice Math Tournament
February 24, 2007
1. Answer: 1
Pick up a piece of fruit in the mixed bin; say it is an apple. Then you know that this bin should be
labeled apples, and the bin currently labeled apples must be oranges (since you know that the
oranges label was incorrect). And so the current oranges should be mixed.

2. Answer: 97
If there are x Lumixians and y Obscrans, x + y = 60 and 4x + y = 129; solving gives x = 23 and
y = 37, so x + 27 = 97.
1
3. Answer: 3
Since there are three red marbles, drawing a red marble means that 2/3 of the time she has drawn
from the box with two red marbles, so only 1/3 of the time is the other marble blue.
4. Answer: 1
Note that sin(90 ) = cos and cos(90 ) = sin ; writing out the tangents as sines divided by
cosines, we see that 1 = tan 10 tan 80 = tan 20 tan 70 = .
4
5. Answer: 3
The arc forming half the boundary of the intersection is of measure 120 , so the circumference is
2 120
360 2.
65
6. Answer: 81
P (lose) = 1 P (lose all 5) = 1 ( 23 )4 = 65
81 .
5
7. Answer: 2
or 2.5
Essentially, the boat is traveling at 4 mph toward a stationary branch 10 miles away. It will take
10/4=2.5 hours to reach it.
3
8. Answer: 8
Let Silas receive a letter from Jessica: then there are 3 ways to arrange the remaining envelopes and
letters by letting Jessica receive letters from either Silas, Katie, or Lekan. The same is true if Silas
receives the letter from Katie or Lekan, so there are 3 3 = 9 total ways for no one to receive the correct
letter. There are a total of 4! ways to receive letters, so the probability we want is 9/4! = 3/8.

9. Answer: 10
0.1(2n 1)
After n days, Peter will have earned 6.4n dollars, while Crocodile will have earned 21 dollars.
0.1(2n 1) n6 1 96
We want the n to satisfy 6.4n < 21 .
Simplifying: n < 2 64 . Since 9 > 2 = 8 but
10 < 2106 = 16, so the smallest such n is 10.
10. Answer: 18
Draw a Venn diagram - we can put the most in the center if none are friends with just two of them.
Let x be the number of students friends with all 3; then 24 x, 20 x, and 39 x are friends with
just Al, Betty, and Clara respectively. Adding up, x + (24 x) + (20 x) + (39 x) = 47, since they
are 3 members of the class of 50, so x = 18.

1
11. Answer: 23520
There are 8! arrangements of the letters, but we divide by 3! and 2! since switching the as or the
rs has no effect. There are then seven ways to split each of these into a first and last name, giving
8! 7/(2!3!) = 8 7 7 5 4 3 possible names.
12. Answer: 5
After nine slices, he has cut 9 30 + 0 + 2 + + 16 = 270 + 18 4 = 342 , so the leftover slice is
18 , with area 100 18 /360 = 5.
13. Answer: 2
The ratio of distances along the rope, ground (shadow), and above the ground is 10 : 6 : 8, so having
travelled 30 5 cm = 1.5m along the ground, it has travelled 8 1.5/6 m = 2 m above the ground.
14. Answer: 148

49
X
(ai+1 ai ) = a50 a1 149 1
j=1

3
15. Answer: 4
We draw the rectangle from 0 to 1 on the x-axis and 1 to 1 on the y-axis; x > y is the region below
a line from the origin to (1, 1). This region has area 23 out of the total area of 2.
16. Answer: 3
Let s be the side length of the square. Looking at one of the 30-60-90 triangles outside the square

would give 2+ 23s 3 = s, so s = 3.
17. Answer: 50%
Let T +/ indicate the test result and B +/ indicate whether the person actually does or does not
have bifurcation virus. The probability that someone has the virus, given that their test is positive,
is equal to the probability that a given person tests positive and has it over the total probability of
testing positive. In statistical notation:
P (T + |B + ) P (B + )
P (B + |T + ) =
P (B + ) + P (T + |B ) P (B )
P (T + |B + )
0.99 0.01
=
0.99 0.01 + (1 0.99) (1 0.01)
= 1/2.

18. Answer: 900


Since L = 1200 2W , Area=W L = W (1200 2W ) = 2W 2 + 1200W . This is a parabola, so its
vertex is at W = 1200/(2 2) = 300, giving L = 600.
19. Answer: a, d, c, b
We know d > a: 100! > 101 00 since each is a product 100 numbers but the first has larger ones. We
know c > d since 1000000! > (100000!)10 > (100!)1 0. Finally, b > c since 1000000! < 10000001000000 =
6 6
(106 )10 = 10610 .
20. Answer: 2n2
There are a total of 4n+2

2 = 8n2 + 6n + 1 pairs of paths; subtracting the pairs which represent
intersections of only two lines, there are 6n2 pairs of paths left. Each point at which three paths
2
intersect accounts for three pairs, so there are at most 6n3 points at which exactly three paths intersect.

2
5111
21. Answer: 5555

105 x = 92007.2007 and 10x = 9.2007


105 x 10x = 92007.2007 9.2007 = 91998
91998 5111
x= =
99990 5555
137
22. Answer: 6
If an event happens with probablity p each second, the expected number of seconds for it to occur is
1/p (this is a definition of probability!). Thus adding up for each dropped ball, the expected time until
no balls are left is 10 10 10 10 10
5 + 4 + 3 + 2 + 1 = 10(
12+15+20+30+60
60 ) = 16 137 = 137
6 seconds.

23. Answer: 11
Note that in a quadrilateral ABCD we have AB + BC + CD > AD, since AB + BC > AC and
AC + CD > AD by the triangle inequality. This gives us three possible equations: 3 + x + y > 3 (this
will always be true), x + 6 > y, and y + 6 > x y > x 6. Thus x + 6 > y > x 6, giving 11 possible
values from x 5 to x + 5.
11
24. Answer: 32
For Andy (A) to win, he must win 4 points before Bob wins 3.
4
P (A wins on 4th point) = 12
1 4 1
 
P (A wins on 5th point) = (Ways to happen) P (A wins 4) P (A loses 1) = 4 2 2
4 2
P (A wins on 6th point) = 52 12 21

4 4 4 2
Sum: 12 + 4 12 12 + 52 12 12 = 22 11
 
64 = 32 .

25. Answer: 12
Let M be the endpoint of the altitude on the hypotenuse. Since we are dealing with right triangles,
M AC ABC, so AM = 12/5. Let N be the endpoint he reaches on side AC. M AC N AM ,
so M N
AM = 4/5. This meansthat each altitude that he walks gets shorter by a factor of 4/5. The total
distance is thus 12 4
5 / 1 5 = 12.

3
RMT 2008 Geometry Solutions February 23, 2008

1. Answer: 10
Label three consecutive vertices of the polygon A, B, and C. Let BP be the common side to the
pentagons placed on sides AB and BC. Then mABP = mP BC = 108 . Since mABP +
mP BC + mABC = 360 , this gives mABC = 144 . So the exterior angle of this polygon is 36,
and thus it has 10 sides.

2. Answer: 8 2
Notice that the ball travels the length of the room twice and the width of the room twice, so its
traveled a total of 8 meters in the horizontal direction and 8 meters in the vertical direction. Because
the ball is bouncing (and thus its path after a bounce is the same as its path before the bounce, but
reflected), we can rearrange the four segments of its path into a straight line by only reflection and
translation. This line travels 8 meters
horizontally and 8 meters vertically, so its length, which is the
total length of the balls path, is 8 2.

3
3. Answer: 2

Since the space diagonal of the cube is a diameter of the sphere, we have s 3 = 2r. The ratio is then
4 3

3 r 3
 3 =
2r

2
3


4. Answer: 288 3 432
Let r be the radius of a small circle. The centers of the smallcircles form an equilateral triangle of
side length 2r. The length of the median of such a triangle is 3r, so the
distance from the center of
the triangle (which is also the center of the large circle) to a vertex is 2 3 3 r. Since each vertex of the

triangle is distance r from the edge of the large circle, the radius of the large circle is 2 3 3 r + r = 144.

233

This gives (2 3 + 3)r = 432, so r = 2432
3+3 2 33
= 144(2 3 3) = 288 3 432.

5. Answer: 15 + tan1 x or
12
+ tan1 x

2 3+x
From basic trigonometry, we have tan(mB) = 1(2
3)x
. This is the tangent angle addition identity,


for angles with tangents x and 2 3. Since tan(15 ) = 2 3, mB, the inverse tangent, is therefore
15 + tan1 x.
44
6. Answer: 13
5
Let = mE and = mF . Note that D is a right angle. Therefore, sin = 13 . [CBF ] =
1 1 5 1 1 1 12
2 11 2
sin = 2 121 13 . Similarly, [ABE] = 2 22
sin = 2 4 cos = 2 4 13 . Finally, [ACD] = 31
2 .
Subtracting these three areas from that of 4DEF gives the result.
7. Answer: 20082 or 4032064
Note that we can scale the triangle down by a factor of 2008 to a 3,4,5 right triangle. Let AB, AC be
the legs of the triangle. The incircle splits AB into two segments of lengths x and y. It similarly splits
AC into segments of lengths x and z and BC into segments of lengths y and z. Thus, we get:

x+y =3
x+z =4
y+z =5

Thus, x = 1, y = 2, z = 3. Thus, the incircle has a radius of 1, and so an area of . Scaling back up
will increase the incircles radius by a factor of 2008, giving us an area of 20082 .
RMT 2008 Geometry Solutions February 23, 2008


30
8. Answer: 3
Let O be the center of the circle, and X be the center of the rhombus (the intersection of AC and
BD). Let mABC = = cos1 23 . Considering 4OBX and 4ABX, using triangle angle sums
and the fact that an inscribed angle has half the measure of the intercepted arc, we have OX =
cos( ), so AX = 1 +q cos( ). Also, BX = sin( ). The Pythagorean theorem then gives
p
l = 2(1 + cos( ) = 2(1 + 32 ).

9. Answer: 12
Let the trapezoid be ABCD with AB = 10, CD = 15. Let P be the intersection of the diagonals,
and let XY be the segment through P parallel to the bases with X on AD and Y on BC. Note that
4P Y C 4ABC, so PAB Y
= YBC
C
. Also, 4P Y B 4DCB, so CD PY
= BY
BC . Adding these equations gives
PY PY BY +Y C 1 1 1
AB + CD = BC = 1, so P Y ( 10 + 15 ) = P Y 6 = 1, hence P Y = 6.
The same argument shows that P X = 6, so XY = 12.
10. Answer: 3
The polygon has angles of 171 , and the smallest triangle has two adjacent sides of the original polygon
as two of its sides. The area of this triangle is 21 1 1 sin(171) = 21 sin(9). So the question is,

how many square roots do we need to express sin(9)? Conveniently enough, sin(18) = 51 4 , so
q
2
cos(18) = 1 sin (18), which requires two square roots to express. Then by the half-angle formula,
q
sin(9) = 1cos(18)
2 , which requires three square roots.
RMT 2009 General Solutions February 21, 2009

1. Answer: 17
Let the answer be v. The raptors gain 20(v 10) while the train is moving, then 13v after it stops, for
a total of 33v 200. In this time they have gained 595, so we have 395 = 35v 200, so v = 17.
2. Answer: 350
We see that since this is at the corner of the barn, the snake is free to travel in a three quarter arc
around this point such that it does not intersect the barn. On the corners, we notice that the leash
will bend, and basically act as a shorter leash fixed at the other two corners. Thus, there is a leash of
length 10 for each of two quarter circles, which sum to half of an arc:
3 1
(20)2 + (10)2 = 350.
4 2
5
3. Answer: 4
This can be expanded, and we see that it is quadratic:
b 5
(2x + 15)(10 x) = 2x2 + 5x + 150 = = =x
2a 4

4. Answer: 144
The two different angles in a parallelogram sum to 180 . If x is the smaller angle, we have x + 4x =
5x = 180 , so x = 36 and the larger angle is 4x = 144 .

5. Answer: 15501
Each upgrade will reduce the total costs by $1000.1. Therefore the first ten upgrades, up to the one
that costs $1000, will save money. The cost of the fizzbangs will then be one dollar each, costing
$10001, and the upgrades will cost a total of $5500.
6. Answer: 1
Suppose the point (x, y) is the intersection point. The distance from (y, x) to the origin is the same,
and it will also be on the line y = 10 x. The slope of the line from the origin to (y, x) is 1/m,
providing a second solution. The product of these two solutions is 1.
7. Answer: 69
140 140 140
The number of tarts made is 140, the least common multiple of 4, 7, and 10. This uses 4 + 7 + 10 =
35 + 20 + 14
225
8. Answer: 64
3 15
The cone of melted ice cream is similar to the original cone, so it has a radius of 8 5 = 8 . The volume
2
of the cone is therefore 31 3 15
8 .
9. Answer: 13
Let x be the length of wall without windows. The length of wall with windows is 20 x, so the total
number of bricks used is 2120 = 120x + 80(20 x). Solving for x gives x = 21201600
40 = 13.
10. Answer: 49
Let x be the normal number of 3-button mice, and y the normal number of 5-button mice. We have
3x + 5y = 207 and 5x + 3y = 281. Subtracting 3 times the first equation from 5 times the second gives
25x 9x = 5 281 3 207, so 16x = 784 and x = 49.

11. Answer: 61
Notice that 3599 = 602 12 . Factoring the difference of two squares gives 602 12 = (60 1)(60 + 1).
59 and 61 are both prime, so the answer is 61.
RMT 2009 General Solutions February 21, 2009

12. Answer: 8
The number of factors of pn1 1 pn2 2 . . . pnmm , where each of pi is a prime, 1 i m, is

(n1 + 1)(n2 + 1) . . . (nm + 1).

Since 12345 = 3 5 823, we have (1 + 1)(1 + 1)(1 + 1) = 8 factors.


13. Answer: 14
We have three different colors, but each glove must have a matching pair of gloves. Thus, there are
8 12 6
2 = 4 white lefthanded gloves, 2 = 6 black lefthanded gloves, and 2 = 3 gray lefthanded gloves. We
have 4 + 6 + 3 = 13. But pulling out one more glove ensures a pair since all the remaining gloves are
righthanded. Thus, we need 14 gloves to make sure we have a matching pair.
1
14. Answer: 2
The first 19 flips are irrelevant. The probability that the last flip will be heads is still 12 .
15. Answer: 273
Nathan can proves theorems at a rate of 61 theorems per hour, while the Lemm-o-Matic 1729 can prove
theorems at a rate of 15 theorems per hour. Working together, Nathan and the Lemm-o-Matic can
prove 15 + 16 = 11
30 theorems per hour. To prove 100 theorems, it will Nathan and the Lemm-o-Matic a
100
total of 11/30 273 hours.

16. Answer: 0
There are 0 such values of N . By kicking out one of their own members, the Silly party can at most
reduce the number people needed to pass a bill by one, but at the same time, they reduce the number
of voters in the Silly party by one. Thus, the number of votes needed from the Sensible party does not
decrease.
17. Answer: 1
You can see immediately that 1 is a root, so factor the equation as (x 1)(x3 2x2 + 2x 1). The
second factor still has 1 as a root, so factor it further as (x 1)2 (x2 x + 1). The second factor now
has no real roots. The answer is 1, since 1 is the only real root.
18. Answer: 500000500000
Pair the terms up: 1 pairs with 1000000, 2 pairs with 999999, etc. There are 500000 pairs, each of
which sums to 1000001, so the sum is 500000500000.
19. Answer: 5
Suppose we have just three triangles. The only way to join them (up to rotations and reflections) is to
make an isosceles trapezoid. If we add a fourth triangle to this, we can get at most six sides. However,
with a fifth triangle, we can get 7 sides. The arrangement that works comes from making a regular
hexagon out of six triangles, then removing one of them.

20. Answer: 2 3
The focus, at points (c, 0) and (c, 0), satisfy the equation a2 b2 = c2 where a and b are the lengths
2 2 2
semiminor axes. Thus, we have that c = 2 1 = 3. The distance between
of the semimajor and the
the two foci are then 2 3.
21. Answer: 1
The expression factors as (n2 n + 1)(n2 + n + 1). If it is to be prime, one of these must equal 1. But
the only way that can happen, for positive integers n, is if n = 1.
189
22. Answer: 256
Probability is independent, so the probability
 that any given  coin is eaten is 1 (1/2 1/2) = 3/4.
The probability of getting 3 or 4 coins is 43 (3/4)3 (1/4)+ 44 (3/4)4 = 108/256 + 81/256 = 189/256.
RMT 2009 General Solutions February 21, 2009

54
23. Answer: 11
minutes
The area of pizza Cody, Frank, and Jeffrey can eat per minute are , 2 , 3 , respectively. So together,
they can eat + 2 + 3 = 11 6 54
6 square meters of pizza per minute. Thus, it takes 11 9 = 11 minutes
to eat a pizza of area 9.
24. Answer: 10076
The answer is the number of times 8! = 40320 is divisible by 5. This is equal to b 40320 40320
5 c + b 25 c +
40320
b 125 c + ..., which is 8064 + 1612 + 322 + 64 + 12 + 2 = 10076.
RMT 2010 Geometry Solutions February 20, 2010

Note: Figures may not be drawn to scale.


1. Answer: (13, 16, 18)
The normal to the plane is in the direction < 3, 4, 5 > and so the line going through the point
perpendicular to the plane is (11 3t, 16 4t, 22 5t) which intersects the plane at t = 4 and hence
the reflection of the point occurs at t = 8, since the original point is at t = 0.

2. Answer: 107736 or (134)2 6


Translate by 2010 to get (402, 0, 0), (0, 402, 0), (0, 0, 402), then scale by 1/402: (1, 0, 0), (0, 1, 0), (0, 0, 1).
Notice that these three points define an equilateral triangle so the center of the circle defined by the 3
points is the circumcenter, which is also the incenter. The incenter of this triangle is ( 1 1 1
3 , 3 , 3 ), so the
radius of the scaled down circle is
s 2  2  2
1 1 1 6
0 + 1 + 0 = .
3 3 3 3

6

The radius of the original circle is 402 3 = 134 6. The area is then (134 6)2 = 107736.

7
3. Answer: 2
q
15 15 7 5 3
First, use Herons Formula to find the area. The semiperimeter is s = 2 , so the area is 2 2 2 2 =

15 7 A 7
4 . Now the area is equal to the inradius times the semiperimeter, so r = s = 2 .

5 2
4. Answer: 3

The lengths of the sides of the large cube containing the cubeoctahedron are 2, so
thevolume
 of
the
1 1 1 2 2 2 2
containing cube is 2 2. The volumes of the removed pyramids are 3 BH = 3 2 2 2 2 = 24 .

Because there are 8 pyramids removed, the total volume removed is 8 242 = 2
3 . Thus, the total volume

of the cubeoctahedron is 2 2 32 = 5 3 2 .

5. Answer: 15
By angle bisector theorem, AB BD 3 BD
AC = DC 9 = 8BD 24 3BD = 9BD. This implies BD = 2 and
DC = 6. Now draw altitude AE and let x = DE and h = AE. Then by Pythagorean theorem, (BD
DE)2 + AE 2 = AB 2 (2 x)2 + h2 = 9. Similarly, AE 2 + (DE + DC)2 = AC 2 h2 + (x + 6)2 = 81.
Expanding, 9 = (2x)2 +h2 = 44x+x2 +h2 and 81 = h2 +x2 +12x+36.
Subtracting
p the two equations,

we get 16x + 32 = 72, or x = 5/2. Then h2 = 35/4, and AD = x2 + h2 = 25/4 + 35/4 = 15.

9
3

B D x E C


25 13
6. Answer: 3

Let N be the opposite point of M in the circle. Then M N = 50 and N B = 502 302 = 40 from
that 4M BN is right triangle. Let C be the midpoint of AB, then 4M CB and 4M BN are similar,
MB MB
so BC = N B M N = 24, M C = M B M N = 18. Let L be the intersection of AC and the tangent.
Since we have AB and OT parallel, CL = OT = 25,so BL = 1. Since 4M CB 4BLD, we have

BD = M B MBL 5
C = 3 , so M D = M B 2 + BD2 = 25 3 13 .
RMT 2010 Geometry Solutions February 20, 2010

7. Answer: 24
Let v, e, t, q be the number of vertices, edges, triangular faces, and quadrilateral faces respectively.
Note that each vertex is shared by exactly one quadrilateral, and a quadrilateral provides four vertices.
By simple counting we get v = 4q. Apply the same thing to triangular face, then we have 4v = 3t.
Meanwhile from each vertex we have 5 edges coming out, so 5v = 2e. Thus we have

q = 1/4v, t = 4/3v, e = 5/2v.

And from the Eulers formula v e + (t + q) = 2, we have (1 5/2 + 1/4 + 4/3)v = 1/12v = 2, v = 24.
p
8. Answer: R2 csc2 x2 R cot
Consider the following diagram, where the sphere has radius r:

R R

Note that d = tan , r = sin , and h = r2 x2 d. Plug in r and d gives the above answer.

9. Answer: 8 3
The center of the sphere is located at the centroid of the tetrahedron, which is located 14 of the way up
the altitude from a face to the opposite vertex. In
other words, the tetrahedron has height 4. Let its
edge length be s. Then the altitude of a face is s 23 , and the distance from the centroid of a face to a

vertex is 32 of that, which is 33 . This length and the height of the tetrahedron form a right triangle,
with an edge as the hypotenuse. That is, 31 s2 + 16 = s2 . Thus s2 = 24, and so the area of a face is

6 3. The volume is 13 6 3 4 = 8 3.
10. Answer: 6029.
If we consider a single 1 1 square, and find two regions within it on which the center of the coin of
radius 14 can land the center ( 21 12 ), of area 14 , and the outside edge, where an overlap will occur,
of area 34 .
The total area that the center of the coin can land on is thus
40192
  
1 1
2010 2010 = .
2 2 4
20102
Thus, the probability is 40192 , so a + b = 6029.
RMT 2011 Geometry Solutions February 19, 2011


1. Answer: 2 + 2 2
The path made generates a regular octagon with side length 1, since the exterior
angle of the octagon is
45 deg. Notice that by inscribing
the octagon in a square
of side length 1 + 2, we can easily calculate
that the octagon has area (1 + 2)2 4 14 = 2 + 2 2.



33 3
2. Answer: 28
Looking at cyclic quadrilaterals ABCD and ACDF tells us that mACD = mADC, so 4ACD is
equilateral and mDEA = 120 . Now, if we let mEAD = , we see that mCAB = 60 =
mACB = = 4AED = 4CBA. Now all we have to do is
calculate side lengths. After creating
some 30 60 90 triangles, it becomes evident that AC = 3. Now let AB = x, so BC = 2x.
By applying the Law of Cosines to triangle ABC, we find that x2 = 73 . Hence, the desired area

( 3)2 3
(ABCDE) = (ACD) + 2(ABC) = 4 + 2 21 (x)(2x)(sin 120 ) = 33 3
28 .

3. Answer: 15
After some angle chasing, we find that mDBF = mDF B = 75 , which implies that DF = DB.
Hence the desired perimeter is equal to AF BF + AE + F E = 20 BF + F E.

By the law of sines, sinF 30
E 10
= sin 75 = F E =

5
6+ 2
= 5 6 5 2.
4

Now, to find BF , draw the altitudefrom O to AB intersecting AB atP . This forms a 30 60 90


triangle, so we can see that AP = 5 3/2 = 10BF = BF = 10 5 3. Hence, the desired perimeter
2
is 20 + (5 6 5 2) (10 5 3) = 10 5 2 + 5 3 + 5 6, so the answer is 10 5 + 5 + 5 = 15.

4. Answer: 5 + 19
Rotate the figure around A by 60 so that C coincides B. Let B 0 , C 0 , D0 , E 0 be the points corresponding
to B, C, D, E in the rotated figure. Since E 0 AD = E 0 AC 0 + C 0 AD = EAC + BAD = 30 =
EAD, E 0 A = EA and DA= D0 A, one has E 0 D = ED. So BC = BD + DE + EC can be found if
0 0 0 B 2 + BD 2 2 E 0 B BD cos 120 =
we know
E D. But E D = E 19, so BC = 2 + 19 + 3 =
5 + 19.
A
B0
D0

E0

B = C0 D E C

5. Answer: 10 2
RMT 2011 Geometry Solutions February 19, 2011

We have 4ADE 4CBE, and their length ratio is AD : CB = 1 : 2. Let AE = p and DE = q.


Then we have AB = BE AE = 2DE AE = 2q p and CD = 2p q. Solving for p and q, we have
p = 4 and q = 5. Similarly we have F C = 8 and F D = 10. Let B = . Then F DE = . Apply
the Law of Cosines to 4EBF to get
EF 2 = BE 2 + BF 2 2BE BF cos = 102 + 202 2 10 20 cos = 500 400 cos
and to 4EDF to get
EF 2 = DE 2 + DF 2 + 2 DE DF cos = 52 + 102 2 5 10 cos = 125 + 100 cos .
Solving for EF 2 , we get EF 2 = 200.
6. Answer: tan1 ( d1 ) (or /2 + tan1 d) or other equivalent form

Construct points C1 , C2 , C3 , on l1 progressing in the same direction as the Ai such that C1 =


A1 and Cn Cn+1 = 1. Thus we have C1 = A1 , C3 = A2 , C5 = A3 , etc., with C2n1 = An in
general. We can write Ai Bi Ai+1 = C2i1 Bi C2i+1 = Ci Bi C2i+1 Ci Bi C2i1 . Observe that
4Ci Bi Ck (for any k) is a right triangle with legs of length d and k i, and Ci Bi Ck = tan1 ki
d . So
Ci Bi C2i+1 Ci Bi C2i1 = tan1 i+1 d tan1 i1
d . The whole sum is therefore
 
X i+1 i1
tan1 tan1
i=1
d d

which has nth partial sum


n+1 n 1
tan1 + tan1 tan1
d d d
RMT 2011 Geometry Solutions February 19, 2011

so it converges to tan1 d1 .

7. Answer: 5

0 0
Rotate triangle AP B around A by 90 degrees as in the given figure.
Let P and B be the rotated
0 0 0
images of P and B respectively. Then we have B P = BP , P P = 2AP so

2AP + BP + CP = CP = P P 0 + P 0 B 0 CB 0 = 5.


8. Answer: 6
Consider the cube to be of side length 2 and divide the answer by 4 later. Set the coordinates of the
vertices of the cube to be (1, 1, 1). Then the plane going through an equilateral triangle can be
described by the equation x+y+z = 1. The distance to the plane from the origin is 13 , as (1/3, 1/3, 1/3)
q q
is the foot of the perpendicular from (0, 0, 0). Thus the radius of the circle is 1 ( 13 )2 = 23 , so
the area is 23 . In the case of the unit cube we should divide this by 4 to get the answer
6.
49
9. Answer: 390

First we shall find area(4ADC) 0 0 0 1 0


area(4ABC) : Since 4A RB CC B and A B = 5 BC, it follows that A R = 5 CS.
1

Then area(4AA0 C 0 ) = 21 A0 R AC 0 = 12 ( 15 CS)( 13 AB) = 15


1 1 1
( 2 CS AB) = 15 area(4ABC). Similarly,
area(4AA0 C 0 )
area(4AA0 C) = 21 AP A0 C = 12 AP ( 45 BC) = 45 area(4ABC). So 1
area(4AA0 C) = 12 . Since 4AA C
0 0
0 0
and 4AA0 C share the same base, QC = 12 . Since 4C 0 T D 4CQD, CD = 12 . Using similar
C T 1 C D 1

arguments, since AC 0 = 31 AB, area(4AC 0 C) = 31 area(4ABC). Since CD = 12 0


13 C C, area(4ADC) =
12 0 12 1 area(4BF C)
13 area(4AC C) = 13 3 area(4ABC). Using the same technique, we can find area(4ABC) and
1 1
area(4AEB) area(4CC 0 B 0 ) 23
0

area(4ABC) . We will just briefly outline the remaining process: area(4CC 0 B) = 2 = 14 . So BBFF =
3
1 0 1 4 0 2
4 . Then area(4BB C) = 2 area(4ABC), so area(4BF C) = 5 area(4BB C) = 5 area(4ABC).
RMT 2011 Geometry Solutions February 19, 2011

1 1
area(4BB 0 A0 ) 52
0
Likewise, area(4BB 0 A) = 1 = 51 . So AAEE = 15 . Then area(4AA0 B) = 1
5 area(4ABC), so
2
area(4AEB) = 65 area(4AA0 B) = 16 area(4ABC).
Then area(4DEF ) area(4ADC) area(4BF C) area(4AEB) 12
area(4ABC) = 1 area(4ABC) area(4ABC) area(4ABC) = 1 39 5
2
1
6 = 49
390 .

10. Answer: 2 43

q
2 2

We claim that in general, the answer is 3 (a + b2 + c2 + 4 3S) , where S is the area of ABC.
Suppose that P QR is an equilateral triangle satisfying the conditions. Then BP C = CQA =
ARB = 60 . The locus of points satisfying BXC = 60 is part of a circle Oa . Draw Ob and Oc
similarly. These three circles meet at a single point X inside the triangle, which is the unique point
satisfying BXC = CXA = AXB = 120 . Then the choice of P on Oa determines Q and R:
those two points should also be on Ob and Oc respectively, and line segments P CQ and P BR should
form sides of the triangle. Now one should find the maximum of P Q under these conditions. Note that
BP X and BRX do not depend on the choice of P , so triangle P XR has the same shape regardless
of our choice. In particular, the ratio of P X to P R is constant, so P R is maximized when P X is the
diameter of Oa . This requires P Q, QR, RP to be perpendicular to XC, XA, XB respectively.
From this point there may be several ways to calculate the answer. One way is to observe that
P Q = 23 (AX + BX + CX) by considering (P QR) = (P XQ) + (QXR) + (RXP ). AX + BX + CX
can be computed by the usual rotation trick for the Fermat point: rotate 4BXA 60 around B
to 4BX 0 A0 . Observe that 4BXX 0 is equilateral, and so A0 , X 0 , X, and C are collinear. Hence,
A0 C = AX + BX + CX, and we can apply the Law of Cosines to 4A0 BC to getthat A0 C 2 = c2 + a2
2ac cos (B + 60 ) = a2 + c2 + 2ac sin 60 sin B 2ac cos 60 cos B = a2 + c2 + 2S 3 12 (a2 + c2 b2 ) =
a2 +b2 +c2
q
2 + 2S 3 = P Q = 23 (a2 + b2 + c2 + 4 3S) (where S is again the area of ABC). Plugging

in our values for a, b, and c, and using Herons formula to find S = 10 5 3 2 = 10 3, we can
calculate P Q = 2 43.
RMT 2012 Geometry Test Solutions February 18, 2012

3
1. Answer: 10
Solution: Let O be the center of the circle. Note that CO bisects AB, so the areas of 4ACO and
4BCO are equal. Hence, the desired difference in segment areas is equal to the difference in the areas of
_
2
the corresponding sectors. The sector corresponding to AC has area 5 , and the sector corresponding to
_
3
BC has area 10 , so the desired difference is .
10
2. Answer: 252
Solution: We can use the standard method of setting up a two-variable system and solving for the height
of the trapezoid. However, since one base is half the length of the other, we may take a shortcut. Extend
AB and CD until they meet at E. Clearly, BC is a midline of triangle EAD, so we have EA = 2BA = 26
and ED = 2CD = 28. The area of EAD is therefore four times that of a standard 13-14-15 triangle,
which we know is 21 14 12 = 84 (since the altitude to the side of length 14 splits the triangle into
9-12-15 and 5-12-13 right triangles). The area of the trapezoid is 34 the area of EAD by similar triangles,
and is therefore 3 84 = 252 .
A similar solution notices that after dropping perpendiculars from B and C to AD, we are left with a
rectangle and two triangles that sum to a 13-14-15 triangle.

3. Answer: 3/2
Solution: Let x be 1/4 the area of ABC, and y the area of a 60 degree sector of Oa minus x. Note that

Sa = Sb = Sc = 3x + y, S = x + 3y,

so Sa + Sb + Sc S = 8x = 2|4ABC| = 3/2 .

503
4. Answer:
6
Solution: Let (XY Z) denote the area of triangle XY Z.
After a bit of angle-chasing, we can use SAS congruence to prove that 4DEF
= 4BF E, so EB
= DF

and therefore AE = F C. If we draw altitudes from E and F onto CD and AB, respectively, we note
that 2AE = 2F C = DF = BE, so AE = 31 AB.
CX CD CY CF
Next, note that 4AEX 4CDX, so AX = AE = 3. Also, 4CF Y 4AEY , so AY = AE = 1. Hence,
XY = 14 AC = (EXY ) = 14 (EAC).
1 1
Finally, AE = 3 AB= (EAC) = 3 (BAC). Since BAC is half the rectangle and therefore has area
503
1006, we get (EXY ) = 1006
12 = .
6

5. Answer: 6/6
Solution: It is obvious that the sphere must be tangent to each face, because if not, then it can be moved
so that it is tangent to four faces; now the radius can be increased until the sphere is tangent to the other
four. Additionally, it is clear that the center of the sphere should be in the center of the octahedron.
Now notice that the sphere must be tangent to the octahedron at the centroid of each face. This can be
seen by symmetry. It is clear that it should be tangent somewhere along the median from one vertex to
the opposite side, and this is true for all three medians, which meet at the centroid.
Now we can proceed in a few ways. One way is to isolate one half of the octahedron i.e. a square-based
pyramid. Slice this pyramid in half perpendicular to the square base and parallel to one of the sides of
the square base. This slice will go through the medians of two opposite triangular faces, in addition to
the center
of the sphere itself. Hence, we get an isosceles triangle ABC with base BC = 1 and legs of
length 3/2. O, the center of the sphere, is the midpoint of BC. If the altitude from O to AB intersects
AB at D, then we have
OD AB = AO BO,
RMT 2012 Geometry Test Solutions February 18, 2012

AOBO

1/ 21/2

since both equal twice the area of AOB, and so DO = AB =
3/2
= 6/6 .

Alternatively, note that our octahedron can be obtained by reflecting the region x + y + z 1/ 2,
x, y, z 0 by xy, yz, zx plane. The inscribing
sphere has itscenter at origin, so its radius is the distance
from the origin to the plane x + y + z = 1/ 2, which is 1/ 6.

6. Answer: 2/3 + 3 1
Solution:
First, it is clear that all of face ABCD can be painted black. This has area 1.
Now we look at the other two visible faces. By symmetry, we only need to consider one of these faces, say
BCGF . Unfold BCGF along BC so that it is coplanar with ABCD, forming a rectangle AF 0 G0 D with
width 1 and height 2. Now, it is clear that the region that can be painted on BCGF is precisely the part
of BCG0 F 0 that is at most two units away from A. Let a circle centered at A with radius two intersect
DG0 at X. Since AX = 2, AD = 1, and AD XD, we conclude that DAX = 3 = F 0 AX = 6 .
Letting (P1 P2 . . . Pn ) denote the area of the n-gon with vertices P1 , . . . , Pn , the desired area equals

0 22 3
area of sector F AX + (AXD) (ABCD) = + 1.
12 2

Putting all this together, we get our final answer to be



3 2
1 + 2(/3 + 1) = + 31 .
2 3

6 11
7. Answer: 11
Solution: We claim that the answer is equal to the inradius in general. Let Ta = ABa Ca , Tb = Ab BCb ,
Tc = Ac Bc C be the smaller triangles cut by the tangents drawn to O. Also let D, E, F be the points of
tangency between O and BC, CA, AB respectively. By considering the fact that tangents to O should
have same length we have ABa + Ba Ca + Ca A = AE + AF . If we sum this over all vertices, then we can
see that the sum of the perimeters of Ta , Tb , Tc is the same as the perimeter of A. Then, the Principle of
Similarity gives ra + rb + rc = r where r is inradius of ABC. It can be calculated by Herons formula as
p
s(s a)(s b)(s c) 11 6 3 2 6 11
r= = = .
s 11 11

Alternatively, let ha denote the height of the altitude from A to BC, and let r be the inradius of ABC.
Since 4ABC 4ABa Ca and since the altitude from A to Ba Ca has length ha 2r, we get
ra ha 2r
= .
r ha
Noticing that
(ABC) aha
r= 1 = ,
2 (a
+ b + c) a + b+c
we get
a
ra = r 2r .
a+b+c
Applying the same reasoning to rb and rc , we can compute

ra + rb + rc = 3r 2r = r.
RMT 2012 Geometry Test Solutions February 18, 2012


45 14
8. Answer: 56
Solution: Let the point where all three circles intersect be denoted as X.
First, note that AOb XOc , BOc XOa , and COa XOb are all rhombi. This helps us easily prove that
AOb k BOa . Since these segments are also congruent, we get that ABOa Ob is a parellogram, and hence
AB = Oa Ob . Similarly, BC
= Ob Oc and CA = Oc Oa .
Note that r is the circumradius of triangle Oa Ob Oc at point X. The above congruences show that r is
therefore the circumradius of triangle ABC, which is computed as

abc 356 45 14
= = .
4(ABC) 4 7421 56

39
9. Answer: 5
Solution: Note that ADB and CBD are supplementary. Therefore, we can extend AD past D to a
new point C 0 such that 4DBC
= 4BDC 0 (alternatively, consider flipping 4DBC over the altitude to

BD). Since ABD = AC B, we have 4ABD 4AC 0 B, and so
0

AC 0 AB AB 2 64
= = AC 0 = = .
AB AD AD 5

39
Since AC 0 = AD + DC 0 , we get DC 0 = BC = 64
5 5= .
5

10. Answer: 4 3
Solution: We have ABC = DAB = 120 and BCD = CDA = 60 at the base, and the three
side faces ADE, BCF , and CDG are all equilateral triangles. If those faces are folded down to the
glass plate along AD, BC, and CD respectively, they will form (along with ABCD) a large equilateral
triangle of side length 3. Let E0 , F0 , and G0 be the vertices of this equilateral triangle corresponding to
E, F and G, respectively.
Observe that M , the midpoint of CD, is the centroid of E0 F0 G0 . As side ADE is folded along AD, which
is perpendicular to E0 M , the projection E1 of E (directly downwards onto the glass plate) still lies on
E0 M . This also holds for the projections of F and G, so projections E1 , F1 , and G1 of E, F , and G lie
on E0 M , F0 M and G0 M respectively.
Since EF CD is a rectangle, E1 F1 CD is as well. Thus E1 D is perpendicular to E0 A. Furthermore,
since E0 E1 is perpendicular to AD, we can conclude that E1 should be the center of triangle ADE0 .
Symmetry gives AE = DE = E0 E, so AE0 DE should be a regular tetrahedron. A similar argument
applies to BF0 CF .

The next step is to figure out the location of G. As EG = 3, ED = 1, and DG = 2, it follows that
DEG is right. Similarly CF G is also right, so plane EF G should be perpendicular to plane EF CD.
Now we cut the whole object along the perpendicular bisector plane of AB and consider its cross-section
along the plane. It will cut AB and EF along their midpoints N and P respectively. As ABM P forms
a regular
tetrahedron of side length 1 and N is midpoint of AB, we have N M = N P = 3/2. Also
M G = 3 and M P G is right. Let Qbe the midpoint of M G; since right triangles are inscribed in
semicircles, it follows that P Q = MQ = 3/2. Thus, N P M and QM P are congruent and N P and M G
are parallel. From M G = M G0 = 3 and N P = N M = 3/2, this gives similarity between N M P and
M G0 G, and GG0 = 2P M = 2. Therefore DCGG0 also forms a regular tetrahedron.
Since AE0 DE , BF0 CF , and CG0 DG are all regular tetrahedrons, we have three lines E0 E, F0 F , and
G0 G meeting at a point X where E0 F0 G0 X forms a regular tetrahedron of side length 3. Thus we finally
demystified our object completely: it was obtained by cutting the regular tetrahedron E0 F0 G0 X along
planes EF G, ADE, BCF , CDG. Moreover we find that X is actually our point source,as it is also
directly above M - both the midpoint of CD and the center of E0 F0 G0 - and its height is 6, the same
as that of point source. So the projection of the object to the glass plate will be exactly E0 F0 G0 , an
RMT 2012 Geometry Test Solutions February 18, 2012

equilateral triangle of side length 3. Hence the projection down to the wood plate will give an equilateral

triangle of side length 4, and our answer is its area, 4 3 .
RMT 2013 Geometry Test Solutions February 2, 2013


1. Answer: 51
Solution: Let mA = x and mB = y. Note that we have two pairs of isosceles triangles, so
mA = mACD and mB = mBCD. Since mACD + mBCD = mACB, we have

180 = mA + mB + mACB = 2x + 2y = mACB = x + y = 90 .

Since ACB is right, we can use the Pythagorean Theorem to compute BC as


p
102 72 = 51 .

For a shortcut, note that D is the circumcenter of ABC and lies on the triangle itself, so it must
lie opposite a right angle.

2. Answer: 16 2

Solution: It turns out the rectangle is actually a square with side length 4 2, and hence has

perimeter 16 2 .

3. Answer: 3 + 1 3
Solution 1: Let O be the center of the circle, and let A and B lie on the circle such that
mAOB = 90 . Call M the midpoint of AO and N the midpoint of BO. Let C lie on minor
arc AB such that CM OA, and let D lie on minor arc AB such that DN OB. Finally,
let CM and DN intersect at E. Now, the problem is to find the area of the region bounded by
DE, EC, and minor arc CD.
Notice that ON = 1 and OD = 2, so ON D is a 30-60-90 right triangle. Since DN and AO are
parallel, mN DO = mAOD = 30 . We now see that the area of the region bounded by AM ,
M E, ED, and arc DA can be expressed as the sum of the areas of triangle ON D and sector
AOD minus the area of square M ON E, which evaluates to

1 22 3
1 3+ 1= + 1.
2 12 2 3

Finally, let x denote the desired area. Then, the area of sector AOB is
!
3 22
1+2 + 1 +x= = x = +1 3 .
2 3 4 3

Solution 2: When the pizza is sliced 4 times in both directions, the result is 4 unit squares,
8 congruent approximate quadrilaterals (one edge is curved), and 4 congruent approximate
triangles (again, one edge is curved). Call the area of an approximate quadrilateral x and an
approximate triangle y. Since all these pieces form a circle of radius 2, we get

8x + 4y = 4 4 = 2x + y = 1

Now, consider the long horizontal slice at the bottom of the pizza, consisting of 2 approximate
quadrilaterals and 2 approximate triangles. Define the endpoints of the slice to be A and B.
Define the center of the pizza to be C. Consider the sector of the pizza cut out by AC and BC.
RMT 2013 Geometry Test Solutions February 2, 2013

This is one third of the pizza, as ACB = 120 , and ABC


= BAC = 30 . Therefore, the
area of the sector is 4/3 and the area of triangle ABC is 3. Hence, we get
4
2x + 2y = 3.
3


3

Solving this system gives of equations gives x = 3 1+ 2 and y = 3 +1 3 Therefore, the

smallest piece of pizza has area +1 3 .
3
1
4. Answer: 4
Solution: First, note that the plane also passes through the midpoint of BD by symmetry,
e.g. across the plane containing AD perpendicular to BC. Let M , N , O, and P denote the
midpoints of BA, AC, CD, and DB, respectively. M N = N O = OP = P M = 21 because they
are all midlines of faces of the tetrahedron. Hence, the cross section is a rhombus. Furthermore,
MO = N P because both equal the distance between midpoints of opposite sides (alternatively,
this congruence can be demonstrated by rotating ABCD such that N and P coincide with the
2 1
previous locations of M and O). Hence, M N OP is a square, and its area is 12 = .
4

3
5. Answer: 4
Solution: For any choice of E, we can draw the circumcircle of P EQ. Angle P EQ is inscribed
inside the minor arc of chord P Q, which is of constant length (it must always be the minor
arc because P EQ is clearly always acute). Therefore, maximizing mP EQ is equivalent to
maximizing the measure of minor arc P Q, which in turn is equivalent to minimizing the radius
of the circle.
Hence, we wish to find the smallest circle that intersects ABCD at P , Q, and at least one other
point. A circle of radius 1 can be tangent to sides BC and AD, while a circle with a smaller
radius clearly cannot touch any of the sides of the square. Hence, it is this circle we desire. Let

this circle be centered at O. OP Q is equilateral, so the height from O to P Q has length 23 .
This is also the height from the points of tangency on AD or BC to P Q. E may be either one

3
of these points, resulting in P EQ having area .
4

3 5
6. Answer: 2
Solution: Let the radius of circle P be r. Draw OP , noting that it is perpendicular to AT at
T . Let Q be the point of tangency between circle O and AD. If we drop a perpendicular from
P to meet OQ (extended) at R, then we know that OR = 1 r and OP = 1 + r, so by the

Pythagorean theorem, P R = 2 r. Thus, AQ = 2 r + r.
Let AB be tangent to P at U . By the Two-Tangent Theorem, AQ = AT = AU . Since U B = r,
we have
3 5
(2 r + r) + r = 2 = r = .
2

7. Answer: 68
RMT 2013 Geometry Test Solutions February 2, 2013

Solution 1: First, shift the coordinate system so that the line goes through the origin and the
parabola is now at x = y 2 + 4.
|b|
Let CD lie on the line y = x + b. The distance between lines AB and CD is therefore 2
, which
can be proven by drawing 45-45-90 triangles. This distance is precisely AD = BC, so CD must
also have this length. Hence, the y-coordinates of C and D must have difference |b|
2 , again by
45-45-90 triangles.
Substituting x = y b to x = y 2 + 4 yields y 2 y + (b + 4) = 0. The difference between two
solutions is 1 4(b + 4) = |b|
p 2
2 , which simplifies to b + 16b + 60 = 0. The area of ABCD is
1 2 1
2 b , so we want 2 times the square of the possible values of b as our answer. We can compute
2
this as 16 260
2 = 68 .
Solution 2: Let C = (y12 , y1 ) and D = (y22 , y2 ), and assume without loss of generality that
the points are positioned such that y1 < y2 . Viewing this in the complex plane, we have
B C = (D C)i, so B = (y12 + y1 y2 , y22 y12 + y1 ). Plugging this into y = x + 4 gives us
y2
y22 2y12 + y2 4 = 0. Since AB k DC, the slope of DC is 1, so yy12 y 2 = 1 = y1 + y2 = 1.
1 2
Solving this system ofequationsgives us two pairs of solutions for (y1 , y2 ), namely (1, 2) and
(2, 3). These give 18 and 50 for CD, respectively, so the sum of all possible areas is
18 + 50 = 68 .

8. Answer: 46
Solution: Note that BP = AP + CP . To prove this, form equilateral triangle AP D where D
lies on the extension of CP . Then triangle ACD is congruent to triangle ABP (and can obtained
by rotating triangle ABP by 60 degrees). Therefore, CD = AP + P C = BP . Alternatively,
apply Ptolemys Theorem to cyclic quadrilateral ABCP , which gives BP = AP + CP directly.
Next, apply the Law of Cosines on triangle AP C to deduce that AP 2 + CP 2 + AP CP = 62
(we have used the fact that mAP C = 120 , since it is opposite the 60 angle ABC). Hence,

(AP + CP )2 = 36 + 10 so BP = AP + CP = 46 .

35
9. Answer: 72
Solution 1: First, we make some preliminary observations. Let M be the midpoint of AB and
N be the midpoint of CD. We see that IA and IB lie on isosceles triangle ABN , since AN and
BN are angle bisectors of CAD and CBD, respectively. This shows that AIA and BIB are
coplanar, so they intersect. Moreover, by symmetry, X must lie on M N . Analogous facts hold
for triangle CDM and its associated points: in particular, Y also lies on M N .
Now, we use mass points to determine the location of X on M N 1 . Let an ordered pair (m, P )
denote that point P has mass m. Assume that masses a, b, c, and d at points A, B, C, and D,
respectively, are placed such that their sum lies at X (that is, let X be our fulcrum).
Since
(a + b + c + d, X) = (a, A) + ((b, B) + (c, C) + (d, D)),
it must be that
(b, B) + (c, C) + (d, D) = (b + c + d, IA ),
since IA is the unique point in the plane of BCD and collinear with X and A. This implies that
c = d, since now (c, C) + (d, D) must lie at the midpoint of CD, i.e. N . Now, since X lies on
1
For a rigorous introduction to mass points, we direct the interested reader to http://www.computing-wisdom.
com/jstor/center_of_mass.pdf
RMT 2013 Geometry Test Solutions February 2, 2013

M N , we know (a, A) + (b, B) must lie at M , so a = b as well. Finally, since IA lies on the angle
bisector of BCD, we know that if CIA is extended to intersect BD at a point Z, then
BZ BC 5 b 7
= = = = .
ZD CD 7 d 5
Hence, a suitable mass assignment is a = b = 7, c = d = 5. Now, we have that
((7, A) + (7, B)) + ((5, C) + (5, D)) = (14, M ) + (10, N )
5
is at X, and so M X = 12 M N .
By similar logic, when we pick Y to be the fulcrum, we get masses a = b = 5, c = d = 4, and so
M Y = 94 M N . Hence,
XY 4 5 1
= = .
MN 9 12 36
Finally, to compute M N , we start by noting that
p
CM = 52 22 = 21
by the Pythagorean Theorem in right triangle AM C. Now, looking at right triangle M N C, we
get s  2
7 35 35
M N = 21 = = XY = .
2 2 72

Solution 2: We present a variant of the first solution that does not require using mass points in
three dimensions. Instead, we will use mass points on the triangle ABN . Let X be our fulcrum.
Recall that AXIA are colinear. We need to compute IBI AN
A
, which we can do by the Angle
Bisector Theorem in triangle BCD. Since CXA bisects angle BCD, we have IBI AN
A CB
= CN = 107 .
Therefore, we can assign a mass of 10 to N and 7 to A. By symmetry, B also gets a mass of 7,
so M X 10 5 MY
M N = 7+7+10 = 12 , as before. This computation extends to get M N = 9 .
4

Using these ratios, the final answer can be computed as in Solution 1.



5 1023
10. Answer: 4
Solution: We first prove a lemma. Let M be the midpoint of AB and N be the midpoint of

EF . Then KLM N is a square. We do this using vectors. Let v1 = CA, v2 = BA, u1 = CD,

and u2 = BF . We first calculate w = EF . Then w = (v1 v2 + u2 ) (u1 + v1 ) = u2 v2 u1 .

Now, we calculate CN in two different ways. First, CN = u1 +v1 + w2 = v1 + u22 + u21 v22 . Second,

CN = v1 v22 + M N . Equating these two gives us M N = u2 +u 2 . Taking the dot product of M N
1


with CB = v1 v2 gives v1 u2 v
2
2 u1
, which is zero. In addition, note that u1 , u2 are rotations of
v1 , v2 such that the angle between v1 and v2 is supplementary to the angle between u1 and u2 .

Hence, the length of M N is the same as the length of LM = v1 v 2 . A similar argument on LK
2

gives the same result, and hence KLM N is a square.


Now, we see that LK = 21 BC. Symmetrically, LJ = 21 AB. Furthermore, angle KLJ is sup-
plementary to angle ABC. Hence, the area of triangle JKL is a quarter of the area of triangle
ABC, and so is the area of a triangle with side lengths half those of ABCs. The area of JKL
may thus be calculated with Herons formula:
r
31 15 11 5 5 1023
= .
2 2 2 2 4
RMT 2013 Geometry Test Solutions February 2, 2013

F
u2
w

H
B
G

M
v2
v1 L
N A C
J

u1
E D
RMT 2014 Geometry Test Solutions February 15, 2014

1. The coordinates of three vertices of a parallelogram are A(1, 1), B(2, 4), and C(5, 1). Compute
the area of the parallelogram.
Answer: 18
Solution: Note that the area of the parallelogram is double the area of triangle ABC. If we
take AC as the base of the triangle, the height is 3, so the area is 36
2 = 9. Thus, the area of the
parallelogram is 18 .

2. In a circle, chord AB has length 5 and chord AC has length 7. Arc AC is twice the length of
arc AB, and both arcs have degree less than 180. Compute the area of the circle.
625
Answer: 51
_ _
Solution: Draw B between A and C. Because AC= 2 AB, BC = 5 too. Let M be the
midpoint of AC. Then s  2
7 51
BM = 52 = .
2 2
Drawing a radius to A and applying the Pythagorean Theorem,
 2 !2
2 7 51 25
r = + r = r =
2 2 51

625
so the area is .
51
3. Spencer eats ice cream in a right circular cone with an opening of radius 5 and a height of 10.
If Spencers ice cream scoops are always perfectly spherical, compute the radius of the largest
scoop he can get such that at least half of the scoop is contained within the cone.

Answer: 2 5
Solution: Since the cone is symmetric, the points of tangency of the ice cream scoop to the
cone make a circle on the inside of the cone. Furthermore, the center of the ice
cream scoop must

coincide with the center of the base of the cone. The slant height of the cone is 52 + 102 = 5 5,
so from considering similar triangles within a cross section of the cone, we get
5 r
= = r = 2 5
5 5 10
.

4. Let ABC be a triangle such that AB = 3, BC = 4, and AC = 5. Let X be a point in the


triangle. Compute the minimal possible value of AX 2 + BX 2 + CX 2 .
50
Answer:
3
Solution: Let the perpendicular distance from X to BC and BA be x and y, respectively. Then

AX 2 + BX 2 + CX 2 = x2 + y 2 + (3 x)2 + (4 y)2 + x2 + y 2

4 2
 
50 50
Completing the square gives 3 y + 3(x 1)2 + , which has minimum .
3 3 3
RMT 2014 Geometry Test Solutions February 15, 2014

5. Let ABC be a triangle where BAC = 30 . Construct D in 4ABC such that ABD =
ACD = 30 . Let the circumcircle of 4ABD intersect AC at X. Let the circumcircle of
4ACD intersect AB at Y . Given that DB DC = 10 and BC = 20, find AX AY .
Answer: 150
Solution: Note that ABDX and ACDY are isosceles trapezoids. Thus, AX = DB and AY =
DC. Furthermore, BDC = 360 (360 30 30 30 ) = 90 . Thus, DB 2 + DC 2 = BC 2 =
DB 2 + DC 2 (DB DC)2
400, and DB DC = 10, so AX AY = DB DC = = 150 .
2
6. Let E be an ellipse with major axis length 4 and minor axis length 2. Inscribe an equilateral
triangle ABC in E such that A lies on the minor axis and BC is parallel to the major axis.
Compute the area of 4ABC.

192 3
Answer: 169
Solution: Consider a transformation that scales along the major axis by a factor of 12 so that
the ellipse becomes a circle of radius 1 and the equilateral triangle becomes an isoceles triangle.
Let the transformed triangle be denoted AB 0 C 0 .
0 0
Now, let x = |B 2C | . Then the original length |BC| = 4x and since 4ABC is equilateral,

|AB| = |BC| = 4x. Next, we calculate the altitude
dropped from A0 as 1 + 1 x2 . Applying
Pythagorass theorem, we get that |AB|2 = (1 + 1 x2 )2 + 4x2 . Substitute |AB| = 4x in and
48
solving for x2 , we get that x2 = 169 .
Finally, we calculate the area of 4ABC. Since 4ABC is equilateral, its area is given by


3 2 3 2 2 48 192 3
4 |BC| = 4 16x . Substituting in x = 169 and simplifying, we obtain 169
.

7. Let ABC be a triangle with AB = 13, BC = 14, and AC = 15. Let D and E be the feet of the
altitudes from A and B, respectively. Find the circumference of the circumcircle of 4CDE.
39
Answer:
4
Solution: Let X be the intersection of AD and BE. Since CDX = CEX = 90 , CDXE
is a cyclic quadrilateral, and CX is the diameter of the circumcircle of CDE. Using the Law of
Cosines on C, we get

AB 2 = BC 2 + AC 2 2(AB)(BC)(cos C)

169 = 196 + 225 2(14)(15)(cos C)


196 + 225 169 3
cos C = =
2(14)(15) 5

Therefore 4CDA and 4CEB are similar to a 3-4-5 triangle, and so is 4XDB. Thus we have
CD = 9, BD = 5, and DX = 15 39
4 . By the Pythagorean Theorem we have CX = 4 , so the
circumference of the circumcircle is
39
d = .
4

It is also possible to use Herons Formula to calculate the area of the triangle and then find the
lengths of the altitudes.
RMT 2014 Geometry Test Solutions February 15, 2014


8. O is a circle with radius 1. A and B are fixed points on the circle such that AB = 2. Let C
be any point on the circle, and let M and N be the midpoints of AC and BC, respectively. As
C travels around circle O, find the area of the locus of points on M N .

2 1
Answer: 8
+ 2
+ 4
Solution:

We introduce some auxilliary points which will be useful for describing the solution. It may also
help to refer to the accompanying diagram. Extend AO past O to intersect the circle again at
point D, and extend BO past O to intersect the circle again at point E. Note that ABDE is a
square of side length 2. Also let ` denote the perpendicular bisector of AB.
_ _
Consider what happens when C is on arc AE or BD. Then, segment M N does not cross `. So,
the two regions corresponding to C being on these two arcs are disjoint, so we can calculate
their individual areas and add them up.
As C moves from A to E (for
instance), M moves from A to the midpoint of AE, and hence
2
travels a total distance of 2 units. M N AE always, so this region has the same area as a

rectangle with height 22 and base 21 AB = 22 , i.e.

1 _
_ 2. So, the cases when C lies either on AE or
BD together account for a region of area 1.
Now we consider the other cases. Let P and Q be the midpoints of OA and OB, respectively.
We claim that as C moves around circle O, M traces out a circle of radius 21 centered at P ,
and N traces out a circle of radius 21 centered at Q. As proof, note that M P is the midline of
triangle AOC that is parallel to OC. Since OC = 1, M P = 21 always (alternatively, note that
we have a homothety from CD to M P with ratio 12 ).
_
So, when C lies on DE, we can see that the region covered
by M N consists of a circle segment
of angle 2 broken in the middle by a rectangle of width 22 . The height of this rectangle can be


1 2
quickly computed to be 4 .
Hence, the total area here is
2
 2  2 !
1 1 1 1 1 2 2 2 3
+ = + .
4 2 2 2 2 4 2 16 4 8
RMT 2014 Geometry Test Solutions February 15, 2014

_
The case where C lies on AB turns out to be equivalent. Hence, in total, the desired area is
!
2 3 2 1
1+2 + = + + .
16 4 8 8 2 4

9. In cyclic quadrilateral ABCD, AB


= AD. If AC = 6 and AB
BD = 35 , find the maximum possible
area of ABCD.

Answer: 5 11
Solution: We claim that the area of ABCD is constant, though the shape of ABCD depends
on the radius of the circumscribing circle.
First, by Ptolemys, we have
AB CD + BC AD = AB(BC + CD) = AC BD
BD
= BC + CD = AC = 10.
AB
Now, extend CB past B to a point E such that BE
= CD. Since ABCD is cyclic, ABC and
ADC are supplementary, so ADC = ABE. Hence, 4ADC = 4ABE, and so ABCD has
the same area as ACE.
ACE is isosceles with leg 6 and base CE = CB + BE = CB + DC = 10. Hence, the length of

the altitude to the base is 62 52 = 11, and the area is 5 11 .
10. Let ABC be a triangle with AB = 12, BC = 5, AC = 13. Let D and E be the feet of the
internal and external angle bisectors from B, respectively. (The external angle bisector from B
bisects the angle between BC and the extension of AB.) Let be the circumcircle of 4BDE;
extend AB so that it intersects again at F . Extend F C to meet again at X, and extend
AX to meet again at G. Find F G.
1560
Answer:
119
Solution: Note that DBE = 90 , so DE is the diameter of . Let BC intersect at G0 . Since
_ _
GE = FE = 90 , G0 XBF is an isosceles trapezoid by symmetry over the diameter DE. By the
same symmetry, 4AXC = 4ABC, so AXC = ABC = 180 G0 BF = 180 CXG0 ;
thus A, X, and G0 are collinear, so G = G0 . Note that GBF = 180 ABC = 90 , so F G is
a diameter; thus F G = DE. Now we calculate DE. Since BE is an external angle bisector,
CE BC
=
AE AB
CE BC
=
AC + CE AB
AC BC 65
CE = = .
AB BC 7
Since BD is an internal angle bisector,
CD BC
=
AD AB
CD BC
=
AC AB + BC
AC BC 65
CD = = .
AB + BC 17
RMT 2014 Geometry Test Solutions February 15, 2014

1560
Thus, the answer is DE = CE + CD = .
119
RMT 2015 Geometry Test Solutions February 14, 2015

1. Clyde is making a Pacman sticker to put on his laptop. A Pacman sticker is a circular sticker
of radius 3 inches with a sector of 120 cut out. What is the perimeter of the Pacman sticker in
inches?
Answer: 4 + 6
Solution: The perimeter of a circle with radius 3 in is 2r = 6. The sector cut out decreases
the perimeter by 120 1
360 = 3 of its perimeter and adds in two lines of length 3. Thus, the perimeter
of the sticker is 23 (6) + 2 3 = 4 + 6 .

2. In a certain right triangle, dropping an altitude to the hypotenuse divides the hypotenuse into
two segments of length 2 and 3 respectively. What is the area of the triangle?

5 6
Answer: 2
Solution: Denote the right triangle ABC with hypotenuse BC. Let D be the intersection of
the altitude and BC and let
CD = 2 and BD
= 3. Triangle ACD is similar to triangle ABC
AC
so CD = BC . Thus, AC = BC CD = 5 2 = 10. Triangle ABD is similar to triangle
AC
AB BC

ABC so BD = AB . Thus, AB = BC BD = 5 3 = 15. Therefore, the area of ABC is

1
5 6
2 10 15 = 2
.

3. Consider a triangular pyramid ABCD with equilateral base ABC of side length 1. AD = BD =
CD and ADB = BDC = ADC = 90 . Find the volume of ABCD.

2
Answer: 24
Solution: Let E be the center of equilateral triangle ABC so that DE is the height of the
pyramid. Then AE is the distance from a vertices of equilateral triangle ABC to its centroid,
and so is 23 sqrt3
2 = 13 . Since AD = BD and ADB = 90 , ADB is a 45-45-90 triangle and

hence AD = AB = 1 . Thus, by Pythagoras, DE =
2 2
AD2 AE 2 = 16 . Now, the area of the


3 1 1 3 2
base ABC is 4 so the volume of ABCD is 3 6 4 = .
24
4. Two circles with centers A and B respectively intersect at two points C and D. Given that
A, B, C, D lie on a circle of radius 3 and circle A has radius 2, what is the radius of circle B?

Answer: 4 2
Solution: First, note that by symmetry, ACB = ADB. Next, since A, B, C, D lie on a
circle, the quadrilateral ACBD is cyclic and hence opposite corners ACB and ADB sum to
180 . Therefore, it follows that ACB = ADB = 90 so AB must be the diameter of the
circle containing points A, B, C, D. Since this circle has radius 3, AB = 6. Next, AC is a radius
of circle A so AC = 2 and BC is a radius of circle B. Applying Pythagoras to the triangle ABC,
we have

AC 2 + BC 2 = AB 2
22 + BC 2 = 62
BC 2 = 32

BC = 4 2
RMT 2015 Geometry Test Solutions February 14, 2015

5. Consider two concentric circles of radius 1 and 2. Up to rotation, there are two distinct equilateral
triangles with two vertices on the circle of radius 2 and the remaining vertex on the circle of
radius 1. The larger of these triangles has sides of length a, and the smaller has sides of length
b. Compute a + b.

Answer: 15
Solution 1: Let a equilateral triangle ABC have A lie on the circle of radius 1 and B, C lie on
the circle of radius 2. Since ABC is equilateral and BC is a chord of the circle of radius 2, the
center of the circles and A must lie on the perpendicular bisector of BC. We see that the two
configurations correspond to where A, B, C all lie on the same semicircle and where A, B, C do
not all lie on the same semicircle.
We first solve for the side length when A, B, C do not all lie on the same semicircle. Let O
denote the center of circle and let D denote the midpoint of BC. In addition, let s denote the
side length of ABC. Since A, B, C do not all lie on the same semicircle, we must have O inside
A.

3s
Since ABC is equilateral, it must have height AD = 2 . In addition, we know that BD = 2s ,

3s2
AO = 1, and BO = 2. Thus, DO = AD AO = 2 . Now, applying the Pythagorean
theorem to triangle BDO, we have

BD2 + DO2 = BO2


!2
 s 2 3s 2
+ = 22
2 2

s2 + 3s2 4 3s + 4 = 16

s2 3s 3 = 0

3 3+43
Thus, it follows that s = 2 . The side length of the equilateral triangle is thus the

3+ 15
positive value s = 2 .
Next, suppose A, B, C all lie on the same semicircle. Then O does not lie inside ABC. Again,
let s denote the side length of ABC. We still have AD = 23s , AO = 1, BO = 2, BD = 2s ,

3s+2
but this time DO = AD + AO = 2 . Applying the Pythagorean theorem to triangle BDO
again, we have

BD2 + DO2 = BO2


 s 2 !2
3s + 2
+ = 22
2 2

s2 + 3s2 + 4 3s + 4 = 16

s2 + 3s 3 = 0

3+ 15 3+ 15 3+ 15
So s = 2 . The sum of the two possible side lengths is therefore 2 + 2 =

15 .
Solution 2: Let the smaller triangle be ABC and the larger triangle be A0 B 0 C 0 . Let the center
of the circles with O, and without loss of generality, let A and A0 be coincident. Finally, let B
and B 0 be on opposite sides of the line AO. Then by symmetry we have that lines BB 0 and CC 0
RMT 2015 Geometry Test Solutions February 14, 2015

form a pair of intersecting chords in the circle of radius 2, intersecting at A = A0 . Let the side
length of ABC be a and the side length of A0 B 0 C 0 be b. Draw the diameter OA, intersecting
the radius 2 circle at points X and Y , and use power of a point to see that the power of A = A0
is (AX)(AY ) = 3 1 = 3. Thus, (AB)(A0 B 0 ) = (AC)(A0 C 0 ) = ab = 3.
Now consider the point E where A0 B 0 intersects the circle of radius 1. Drop a perpendicular
from O to the point D on A 0 B 0 . The triangle OA0 D is then a 30-60-90 triangle with hypotenuse

0
of length 1. Thus, A D = 3/2, and A0 E = 3, as AOE is isosceles. Finally, note that
by
0 0 0 0 0
symmetry, BA = EB = a. But since A B = b = AE +EB = 3+a, we have that b = 3+a.

Plugging this in to ab = 3, we solve for a and b and find that a + b = 15

6. In a triangle ABC, let D and E trisect BC, so BD = DE = EC. Let F be the point on AB
AF AG 1
such that = 2, and G on AC such that = . Let P be the intersection of DG and EF ,
FB GC 2
BX
and extend AP to intersect BC at a point X. Find .
XC
2
Answer:
3
Solution: Note that DG happens to be parallel to AB as BD AG 1
DC = GC = 2 . Therefore triangles
DEP and BEF are similar so we have BF = BE = 2 . This implies that DP = BF
DP DE 1 AB
2 = 6 . Next,
BX
triangles DP X and ABX are similar so we have DX = PAB 6
D = 6. Hence, BX = 5 BD = 5 BC
2

2
and XC = BC BX = 35 BC. So we conclude that BX XC = 3 .

7. A unit sphere is centered at (0, 0, 1). There is a point light source located at (1, 0, 4) that sends
out light uniformly in every direction but is blocked by the sphere. What is the area of the
spheres shadow on the x-y plane? (A point (a, b, c) denotes the point in three dimensions with
x-coordinate a, y-coordinate b, and z-coordinate c).

3 2
Answer: 2
Solution: The region in space that is in shadow due to the sphere is a cone. Therefore, the
spheres shadow on the xy plane is the intersection of a cone and a plane, which is an ellipse.
We proceed to compute the major and minor axes of the ellipse.
First, note that since the y-coordinate of the spheres center and the light source both equal
0, one of the axes must lie along the x-axis. The axes of an ellipse are perpendicular to one
another, so the remaining axis must be parallel to the y-axis.
Now, consider projecting everything onto the xz plane (that is, simply disregard the y coordi-
nate). The sphere is projected onto a unit circle centerd at (0, 1), the light source is projected
to the point (1, 4), and the ellipse is projected onto its horizontal axis. Let ABC be the triangle
consisting of the light source A and let B, C be the two ends of the ellipses axis. The circle is
thus the incircle of ABC, and we see that ABC must be a right angle triangle with ABC = 90 .
Let D be the point where the incircle intersects AB, E be the point where the incircle intersects
BC, and F be the point where the incircle intersects AC. Then AD = AF = 3, BD = BE = 1
and CF = CE. By Pythagoras, AB 2 + BC 2 = AC 2 so 42 + (1 + CE)2 = (3 + CE)2 . Solving
for CE, we find CE = 2, so the horizontal axis of the ellipse BC = 3.
Next, we project everything onto the yz plane. This time, the ellipse is projected onto its vertical
axis. Again, let A be the light source and B, C be the endpoints of the ellipses axis. Then ABC
is a isoceles triangle with AB = BC and the unit sphere is projected onto the incircle of ABC.
RMT 2015 Geometry Test Solutions February 14, 2015

If we let D be the intersection of the incircle and AB, E be the intersection of the incircle and
AC, and F be the intersection of the incircle and BC, then we have CE = CF = BD = BF and
AD = AE. Let O denote the center of the incircle. Then OA= 3 and OD = OE = OF = 1.
By Pythagoras, AE 2 + OD2 = OA2 so AE = 32 12 = 2 2. Applying Pythagoras again,
to ACF 2 2 2 2 2 + CF )2 . Solving for CF , we have
, we have AF + CF = AC so 4 + CF = (2 2
CF = 2. Thus, the vertical axis BC is equal to 2 CF = 2 2.

The spheres shadow on the xy plane is hence an ellipse with axes 3 and 2 2 so the area of the


3 2 2 3 2
shadow is 2 2 = .
2

8. Consider the parallelogram ABCD such that CD = 8 and BC = 14. The diagonals AC and
BD intersect at E and AC = 16. Consider a point F on the segment ED with F D = 366 .
Compute CF .
q
148
Answer: 3
Solution 1: First, note that in a parallelogram the diagonals bisect each other so AE = CE =
AC
2 = 8 and BE = DE. Thus, triangle CDE is isoceles with CD = CE = 8. Drop an altitude
CG from C onto DE. Then DG = EG and BG = 3EG. Applying Pythagoras to triangles
CEG and CBG, we have CE 2 EG2 = CG2 = CB 2 BG2 . Thus,

82 EG2 = 142 (3EG)2


8EG2 = 132

66
EG =
2
q
190
and the altitude is CG = CE 2 EG2 = 64 664 = 2 . Now, since F G = DG F D =

66 66 66
EG F D = 2 3 = 6 . Applying Pythagoras to triangle CF G, we have

CF 2 = F G2 + CG2
66 190
= +
36 4
148
=
3
r
148
so CF = .
3
Solution 2: By the parallelogram law,

(AD)2 + (BC)2 + (AB)2 + (CD)2 = (AC)2 + (BD)2


142 + 142 + 82 + 82 = 162 + (BD)2
(BD)2 = 264

BD = 2 66

Thus
2 66
EF =
3
RMT 2015 Geometry Test Solutions February 14, 2015

Let x = CF .
By Stewarts Theorem:

66 2 66 2 66 66
8 8+8 8 = x 66 x + 66
3 3 3 3
64 66 128 66 132 66
+ = x2 66 +
3 3 9
44 66
64 66 = x2 66 +
3
2 44
64 = x +
3
192 44
x2 =
r3
148
x=
3

9. Triangle ABC is isoceles with AB = AC = 2 and BC = 1. Point D lies on AB such that the
inradius of ADC equals the inradius of BDC. What is the inradius of ADC?

15 3
Answer: 8
Solution: Now, let y denote CD and let x denote BD so AD = 2 x. Since the area of a
triangle is equal to its semiperimeter times its inradius and triangle ADC and BDC have the
ADC
same inradius, the ratio of their areas is the ratio of their semiperimeters. Thus, BDC = 4x+y
1+y+x .
AD
However, the ratio of their areas is also equal to the ratio BD . Thus, we have that
4x+y 2x
=
1+y+x x
4x x + xy = 2 + 2x + 2y x x2 xy
2

2y 2xy = 3x 2
3x 2
y=
2 2x

Next, note that cos(ABC) = 14 since the altitude from A bisects BC. Applying the law of
cosines to triangle BDC, we have

y 2 = x2 + 12 2x cos(ABC)
x
= x2 + 1
2
Combining these two equations, we can solve for x:
 2
2x 3x 2
x +1=
2 2 2x
(2 2x)2 (2x2 x + 2) = 2(3x 2)2
8x4 20x3 + 24x2 20x + 8 = 18x2 24x + 8
8x4 20x3 + 6x2 + 4x = 0
4x4 10x3 + 3x2 + 2x = 0
RMT 2015 Geometry Test Solutions February 14, 2015

Now, notice that x = 0 and x = 2 are extraneous solutions so we may



divide out by x and (x2)
2 1 5
to obtain the quadratic 4x 2x 1 which has solutions x = 4 . One solution is negative

1+ 5
so we may discard it and hence we conclude that x = 4 . Plugging in x into the equation

3x2 5
y= 22x , we see that y = 2 .
Now, let r denote the inradius of ADC, which is equal to theq
inradius of BDC. We have that
2
ADC + BDC = ABC. The height of triangle ABC is 22 21 = 215 so the area of

ABC is 21 215 1 = 415 . The area of ADC is its semiperimeter 4x+y
2 times r and the area of
1+x+y
BDC is its semiperimeter 2 times r. Thus, we have that

4x+y 1+x+y 15
r+ r=
2 2 4
15
(5 + 2y)r =
2
15
(5 + 5)r =
2
15
r=
2(5 + 5)

15 3
r=
8

10. For a positive real number k and an even integer n 4, the k-Perfect n-gon is defined to be the
equiangular n-gon P1 P2 . . . Pn with Pi Pi+1 = Pn/2+i Pn/2+i+1 = k i1 for all i {1, 2, . . . , n/2},
assuming the convention Pn+1 = P1 (i.e. the numbering wraps around). If a(k, n) denotes the
area of the k-Perfect n-gon, compute a(2,24)
a(4,12) .

Answer: 5 25 4
2 + 25
4
6
a(k, 4n)
Solution 1: We find a general formula for .
a(k 2 , 2n)
Let P1 P2 . . . P4n be the k-Perfect 4n-gon. Consider the 2n-gon P1 P3 . . . P4n1 , obtained by
taking every other vertex starting with P1 .
For any i, 1 i 2n 2, 4P1 P2 P3 4Pi Pi+1 Pi+2 with a ratio of k i1 : 1, by SAS similarity.
Therefore, Pi Pi+2 = k i P1 P3 for any such i. Similarly, for i with 2n i 4n 2, we have
Pi Pi+2 = k i2n+1 P1 P3 . So, we conclude that P1 P3 . . . P4n1 is similar to the k 2 -Perfect 2n-gon,
by a ratio of P1 P3 : 1.
By the Law of Cosines,
s   r
2 
P1 P3 = 12 + k2 2 1 k cos = 1 + k 2 + 2k cos .
4n 2n

Therefore, the area of P1 P3 . . . P4n1 is


  
1 + k 2 + 2k cos a(k 2 , 2n).
2n
If we remove this 2n-gon from our larger 4n-gon, we are left with 2n similar triangles. Each has

an angle of 2n with incident edges in a ratio of 1 : k. For each i {0, 2, . . . , n 1}, there
RMT 2015 Geometry Test Solutions February 14, 2015

are two such triangles where the edges incident have lengths k 2i and k 2i+1 . We want to relate
the sum of the areas of these triangles to a(k 2 , 2n) somehow.

Let A be a point in the plane, and construct rays AB0 , AB1 , AB2 , . . . , ABn all emanating from

A such that ABi is n radians clockwise with respect to ABi1 . Note that this makes points B0 ,

Bn , and A collinear. Now, for each i {0, . . . , n}, let Ci be the point on ABi that is k 2i units
from A. Consider the n + 1-gon C0 C1 . . . Cn . For each i {0, . . . , n 1}, 4AC0 C1 4ACi Ci+1
with ratio k 2i . This implies that Ci1 Ci = k 2i C0 C1 . Moreover, the similar triangles also give
us that C0 C1 C2 = Ci1 Ci Ci+1 = mC0 AC1 = n for any i {1, . . . , n 1}. This
is sufficient to demonstrate that C0 C1 . . . Cn is similar to half of the k 2 -Perfect 2n-gon, with a
ratio of C0 C1 : 1.
We can compute C0 C1 also by the law of cosines, getting
r   r  
2 4 2
C0 C1 = 1 + k 2 1 k cos = 1 + k 4 2k 2 cos .
n n

Hence, C0 C1 . . . Cn has area



1 + k 4 2k 2 cos

n
a(k 2 , 2n).
2

Our construction of C0 C1 . . . Cn can be thought of as assembling the polygon from the n triangles
C0 AC1 , C1 AC2 , . . . , Cn1 ACn . These triangles are related to the ones left over from our 4n-gon.

For every triangle with edges k 2i and k 2i+1 meeting at an angle 2n , there is a triangle with
edges k 2i and k 2i+2 meeting at an angle n . Since any triangle ABC has area 21 ab sin C, the ratio
of the sum of areas of the triangles from the 4n-gon to the sum of the areas of the triangles we
just created is

 
2 sin 2n 2 sin 2n
 =
k sin n k sin n


(recall that we had 2n triangles in the first set but n triangles in the second set, hence the factor
of 2). Therefore, the total area in the triangles left over from the 4n-gon is

1 + k 4 2k 2 cos n 4 2k 2 cos
   
2 sin 2n sin 1 + k
 a(k 2 , 2n) = 2n n
a(k 2 , 2n).
k sin n k sin n

2

Adding up, we get that



1 + k 4 2k 2 cos
  sin  
a(k, 4n) 2 2n n
= 1 + k + 2k cos +
a(k 2 , 2n) k sin n

2n

Finally, we can plug in k = 2 and n = 6. This gives us


  sin  17 8 cos 
12 6
5+4 cos +
2 sin 6

12

6+ 2 6 2  
=5+4 + 17 4 3
4 4
25 25
= 5 2+ 6.
4 4
RMT 2015 Geometry Test Solutions February 14, 2015

Note: One might expect at first glance that for fixed k and as n goes to infinity, this ratio would
approach (k + 1)2 . In the limit, 2n-gon formed from taking every other vertex of the 4n-gon will
have side lengths that are k + 1-times that of the k 2 -Perfect 2n-gon, and the leftover triangles
look like their area will tend towards zero. However, we can see that their total area actually
2 1)2
tends to (k 2k , which grows faster than (k + 1)2 as k goes to infinity. As k goes to 1 i.e. as
the polygons become regular, this quantity does approach 0.
Solution 2: The first solution gave us a decomposition of the k-Perfect 2n-gon into 2n similar
triangles (renaming k 2 to k). We can use this decomposition to write out an explicit formula for
the area of the k-Perfect 2n-gon.
Recall that we used n similar triangles to construct a polygon of area
1 + k 2 2k cos n

a(k, 2n).
2
Triangle C0 AC1 had AC0 = 1, AC1 = k, and mC0 AC1 = n , so its area was
1   k  
1 k sin = sin .
2 n 2 n
The other triangles were similar, getting bigger in length by a factor of k each time, so the sum
of the areas of the n triangles is
k   k   k 2n 1
sin (1 + k 2 + + k 2(n1) ) = sin
2 n 2 n k2 1 
1 + k 2k cos n
2
= a(k, 2n)
2
k sin n (k 2n 1)

= a(k, 2n) = 2  .
(k 1) 1 + k 2 2k cos n

Hence, we have
4n 1)(k 4 1) 1 + k 4 2k 2 cos
 
a(k, 4n) k sin 2n  (k n 
2
= 2 4n 1)(k 2 1) 1 + k 2 2k cos
.
a(k , 2n) k sin n (k 2n

(k 1) 1 + k 4 2k 2 cos n
 4 
sin 2n
=  .
k sin n (k 2 1) 1 + k 2 2k cos 2n


Plugging in k = 2, n = 6 yields the same answer as before.


Note: This formula recapitulates our earlier finding that this ratio grows as O(k 3 ) as k grows
large.
Aside: It may not be obvious that k-Perfect 2n-gons exist for any integer n and positive real k.
Here we give a constructive proof of their existence. In fact, we prove something stronger: given
any positive real numbers a1 , . . . , an , we construct an equiangular 2n-gon P1 P2 . . . P2n such that
Pi Pi+1 = Pn+i Pn+i+1 = ai for all i {1, . . . , n}.
Start with P1 P2 . . . P2n , a regular 2n-gon with side length a1 . Now, translate the points P3
through Pn+2 by a2 a1 units away from the other half of the points, in the direction parallel
to P2 P3 (if a2 a1 < 0, move them towards the other points). This maintains all angles and
all edge lengths, except that P2 P3 = Pn+2 Pn+3 = a2 now. Now do the same operation on the
points P4 , P5 , . . . , Pn+3 , and so on. In the end, you will have constructed the desired polygon.

Das könnte Ihnen auch gefallen